Систематическая ошибка теория вероятности

Результат любого измерения не определён однозначно и имеет случайную составляющую.
Поэтому адекватным языком для описания погрешностей является язык вероятностей.
Тот факт, что значение некоторой величины «случайно», не означает, что
она может принимать совершенно произвольные значения. Ясно, что частоты, с которыми
возникает те или иные значения, различны. Вероятностные законы, которым
подчиняются случайные величины, называют распределениями.

2.1 Случайная величина

Случайной будем называть величину, значение которой не может быть достоверно определено экспериментатором. Чаще всего подразумевается, что случайная величина будет изменяться при многократном повторении одного и того же эксперимента. При интерпретации результатов измерений в физических экспериментах, обычно случайными также считаются величины, значение которых является фиксированным, но не известно экспериментатору. Например смещение нуля шкалы прибора. Для формализации работы со случайными величинами используют понятие вероятности. Численное значение вероятности того, что какая-то величина примет то или иное значение определяется либо как относительная частота наблюдения того или иного значения при повторении опыта большое количество раз, либо как оценка на основе данных других экспериментов.

Замечание. 
Хотя понятия вероятности и случайной величины являются основополагающими, в литературе нет единства в их определении. Обсуждение формальных тонкостей или построение строгой теории лежит за пределами данного пособия. Поэтому на начальном этапе лучше использовать «интуитивное» понимание этих сущностей. Заинтересованным читателям рекомендуем обратиться к специальной литературе: [5].

Рассмотрим случайную физическую величину x, которая при измерениях может
принимать непрерывный набор значений. Пусть
P[x0,x0+δ⁢x] — вероятность того, что результат окажется вблизи
некоторой точки x0 в пределах интервала δ⁢x: x∈[x0,x0+δ⁢x].
Устремим интервал
δ⁢x к нулю. Нетрудно понять, что вероятность попасть в этот интервал
также будет стремиться к нулю. Однако отношение
w⁢(x0)=P[x0,x0+δ⁢x]δ⁢x будет оставаться конечным.
Функцию w⁢(x) называют плотностью распределения вероятности или кратко
распределением непрерывной случайной величины x.

Замечание. В математической литературе распределением часто называют не функцию
w⁢(x), а её интеграл W⁢(x)=∫w⁢(x)⁢𝑑x. Такую функцию в физике принято
называть интегральным или кумулятивным распределением. В англоязычной литературе
для этих функций принято использовать сокращения:
pdf (probability distribution function) и
cdf (cumulative distribution function)
соответственно.

Гистограммы.

Проиллюстрируем наглядно понятие плотности распределения. Результат
большого числа измерений случайной величины удобно представить с помощью
специального типа графика — гистограммы.
Для этого область значений x, размещённую на оси абсцисс, разобьём на
равные малые интервалы — «корзины» или «бины» (англ. bins)
некоторого размера h. По оси ординат будем откладывать долю измерений w,
результаты которых попадают в соответствующую корзину. А именно,
пусть k — номер корзины; nk — число измерений, попавших
в диапазон x∈[k⁢h,(k+1)⁢h]. Тогда на графике изобразим «столбик»
шириной h и высотой wk=nk/n.
В результате получим картину, подобную изображённой на рис. 2.1.

Рис. 2.1: Пример гистограммы для нормального распределения (x¯=10,
σ=1,0, h=0,1, n=104)

Высоты построенных столбиков будут приближённо соответствовать значению
плотности распределения w⁢(x) вблизи соответствующей точки x.
Если устремить число измерений к бесконечности (n→∞), а ширину корзин
к нулю (h→0), то огибающая гистограммы будет стремиться к некоторой
непрерывной функции w⁢(x).

Самые высокие столбики гистограммы будут группироваться вблизи максимума
функции w⁢(x) — это наиболее вероятное значение случайной величины.
Если отклонения в положительную и отрицательную стороны равновероятны,
то гистограмма будет симметрична — в таком случае среднее значение ⟨x⟩
также будет лежать вблизи этого максимума. Ширина гистограммы будет характеризовать разброс
значений случайной величины — по порядку величины
она, как правило, близка к среднеквадратичному отклонению sx.

Свойства распределений.

Из определения функции w⁢(x) следует, что вероятность получить в результате
эксперимента величину x в диапазоне от a до b
можно найти, вычислив интеграл:

Px∈[a,b]=∫abw⁢(x)⁢𝑑x. (2.1)

Согласно определению вероятности, сумма вероятностей для всех возможных случаев
всегда равна единице. Поэтому интеграл распределения w⁢(x) по всей области
значений x (то есть суммарная площадь под графиком w⁢(x)) равен единице:

Это соотношение называют условием нормировки.

Среднее и дисперсия.

Вычислим среднее по построенной гистограмме. Если размер корзин
h достаточно мал, все измерения в пределах одной корзины можно считать примерно
одинаковыми. Тогда среднее арифметическое всех результатов можно вычислить как

Переходя к пределу, получим следующее определение среднего значения
случайной величины:

где интегрирование ведётся по всей области значений x.
В теории вероятностей x¯ также называют математическим ожиданием
распределения.
Величину

σ2=(x-x¯)2¯=∫(x-x¯)2⁢w⁢𝑑x (2.3)

называют дисперсией распределения. Значение σ есть
срекднеквадратичное отклонение в пределе n→∞. Оно имеет ту
же размерность, что и сама величина x и характеризует разброс распределения.
Именно эту величину, как правило, приводят как характеристику погрешности
измерения x.

Доверительный интервал.

Обозначим как P|Δ⁢x|<δ вероятность
того, что отклонение от среднего Δ⁢x=x-x¯ составит величину,
не превосходящую по модулю значение δ:

P|Δ⁢x|<δ=∫x¯-δx¯+δw⁢(x)⁢𝑑x. (2.4)

Эту величину называют доверительной вероятностью для
доверительного интервала |x-x¯|≤δ.

2.2 Нормальное распределение

Одним из наиболее примечательных результатов теории вероятностей является
так называемая центральная предельная теорема. Она утверждает,
что сумма большого количества независимых случайных слагаемых, каждое
из которых вносит в эту сумму относительно малый вклад, подчиняется
универсальному закону, не зависимо от того, каким вероятностным законам
подчиняются её составляющие, — так называемому нормальному
распределению
(или распределению Гаусса).

Доказательство теоремы довольно громоздко и мы его не приводим (его можно найти
в любом учебнике по теории вероятностей). Остановимся
кратко на том, что такое нормальное распределение и его основных свойствах.

Плотность нормального распределения выражается следующей формулой:

w𝒩⁢(x)=12⁢π⁢σ⁢e-(x-x¯)22⁢σ2. (2.5)

Здесь x¯ и σ
— параметры нормального распределения: x¯ равно
среднему значению x, a σ —
среднеквадратичному отклонению, вычисленным в пределе n→∞.

Как видно из рис. 2.1, распределение представляет собой
симметричный
«колокол», положение вершины которого
соответствует x¯ (ввиду симметрии оно же
совпадает с наиболее вероятным значением — максимумом
функции w𝒩⁢(x)).

При значительном отклонении x от среднего величина
w𝒩⁢(x)
очень быстро убывает. Это означает, что вероятность встретить отклонения,
существенно большие, чем σ, оказывается пренебрежимо
мала
. Ширина «колокола» по порядку величины
равна σ — она характеризует «разброс»
экспериментальных данных относительно среднего значения.

Замечание. Точки x=x¯±σ являются точками
перегиба графика w⁢(x) (в них вторая производная по x
обращается в нуль, w′′=0), а их положение по высоте составляет
w⁢(x¯±σ)/w⁢(x¯)=e-1/2≈0,61
от высоты вершины.

Универсальный характер центральной предельной теоремы позволяет широко
применять на практике нормальное (гауссово) распределение для обработки
результатов измерений, поскольку часто случайные погрешности складываются из
множества случайных независимых факторов. Заметим, что на практике
для приближённой оценки параметров нормального распределения
случайной величины используются выборочные значения среднего
и дисперсии: x¯≈⟨x⟩, sx≈σx.

x-x0σ2=2w⁢(x)σ1=1

Рис. 2.2: Плотность нормального распределения

Доверительные вероятности.

Вычислим некоторые доверительные вероятности (2.4) для нормально
распределённых случайных величин.

Замечание. Значение интеграла вида ∫e-x2/2⁢𝑑x
(его называют интегралом ошибок) в элементарных функциях не выражается,
но легко находится численно.

Вероятность того, что результат отдельного измерения x окажется
в пределах x¯±σ оказывается равна

P|Δ⁢x|<σ=∫x¯-σx¯+σw𝒩⁢𝑑x≈0,68.

Вероятность отклонения в пределах x¯±2⁢σ:

а в пределах x¯±3⁢σ:

Иными словами, при большом числе измерений нормально распределённой
величины можно ожидать, что лишь треть измерений выпадут за пределы интервала
[x¯-σ,x¯+σ]. При этом около 5%
измерений выпадут за пределы [x¯-2⁢σ;x¯+2⁢σ],
и лишь 0,27% окажутся за пределами
[x¯-3⁢σ;x¯+3⁢σ].

Пример. В сообщениях об открытии бозона Хиггса на Большом адронном коллайдере
говорилось о том, что исследователи ждали подтверждение результатов
с точностью «5 сигма». Используя нормальное распределение (2.5)
нетрудно посчитать, что они использовали доверительную вероятность
P≈1-5,7⋅10-7=0,99999943. Такую точность можно назвать фантастической.

Полученные значения доверительных вероятностей используются при
стандартной записи результатов измерений. В физических измерениях
(в частности, в учебной лаборатории), как правило, используется P=0,68,
то есть, запись

означает, что измеренное значение лежит в диапазоне (доверительном
интервале) x∈[x¯-δ⁢x;x¯+δ⁢x] с
вероятностью 68%. Таким образом погрешность ±δ⁢x считается
равной одному среднеквадратичному отклонению: δ⁢x=σ.
В технических измерениях чаще используется P=0,95, то есть под
абсолютной погрешностью имеется в виду удвоенное среднеквадратичное
отклонение, δ⁢x=2⁢σ. Во избежание разночтений доверительную
вероятность следует указывать отдельно.

Замечание. Хотя нормальный закон распределения встречается на практике довольно
часто, стоит помнить, что он реализуется далеко не всегда.
Полученные выше соотношения для вероятностей попадания значений в
доверительные интервалы можно использовать в качестве простейшего
признака нормальности распределения: в частности, если количество попадающих
в интервал ±σ результатов существенно отличается от 2/3 — это повод
для более детального исследования закона распределения ошибок.

Сравнение результатов измерений.

Теперь мы можем дать количественный критерий для сравнения двух измеренных
величин или двух результатов измерения одной и той же величины.

Пусть x1 и x2 (x1≠x2) измерены с
погрешностями σ1 и σ2 соответственно.
Ясно, что если различие результатов |x2-x1| невелико,
его можно объяснить просто случайными отклонениями.
Если же теория предсказывает, что вероятность обнаружить такое отклонение
слишком мала, различие результатов следует признать значимым.
Предварительно необходимо договориться о соответствующем граничном значении
вероятности. Универсального значения здесь быть не может,
поэтому приходится полагаться на субъективный выбор исследователя. Часто
в качестве «разумной» границы выбирают вероятность 5%,
что, как видно из изложенного выше, для нормального распределения
соответствует отклонению более, чем на 2⁢σ.

Допустим, одна из величин известна с существенно большей точностью:
σ2≪σ1 (например, x1 — результат, полученный
студентом в лаборатории, x2 — справочное значение).
Поскольку σ2 мало, x2 можно принять за «истинное»:
x2≈x¯. Предполагая, что погрешность измерения
x1 подчиняется нормальному закону с и дисперсией σ12,
можно утверждать, что
различие считают будет значимы, если

Пусть погрешности измерений сравнимы по порядку величины:
σ1∼σ2. В теории вероятностей показывается, что
линейная комбинация нормально распределённых величин также имеет нормальное
распределение с дисперсией σ2=σ12+σ22
(см. также правила сложения погрешностей (2.7)). Тогда
для проверки гипотезы о том, что x1 и x2 являются измерениями
одной и той же величины, нужно вычислить, является ли значимым отклонение
|x1-x2| от нуля при σ=σ12+σ22.


Пример. Два студента получили следующие значения для теплоты испарения
некоторой жидкости: x1=40,3±0,2 кДж/моль и
x2=41,0±0,3 кДж/моль, где погрешность соответствует
одному стандартному отклонению. Можно ли утверждать, что они исследовали
одну и ту же жидкость?

Имеем наблюдаемую разность |x1-x2|=0,7 кДж/моль,
среднеквадратичное отклонение для разности
σ=0,22+0,32=0,36 кДж/моль.
Их отношение |x2-x1|σ≈2. Из
свойств нормального распределения находим вероятность того, что измерялась
одна и та же величина, а различия в ответах возникли из-за случайных
ошибок: P≈5%. Ответ на вопрос, «достаточно»
ли мала или велика эта вероятность, остаётся на усмотрение исследователя.

Замечание. Изложенные здесь соображения применимы, только если x¯ и
его стандартное отклонение σ получены на основании достаточно
большой выборки n≫1 (или заданы точно). При небольшом числе измерений
(n≲10) выборочные средние ⟨x⟩ и среднеквадратичное отклонение
sx сами имеют довольно большую ошибку, а
их распределение будет описываться не нормальным законом, а так
называемым t-распределением Стъюдента. В частности, в зависимости от
значения n интервал ⟨x⟩±sx будет соответствовать несколько
меньшей доверительной вероятности, чем P=0,68. Особенно резко различия
проявляются при высоких уровнях доверительных вероятностей P→1.

2.3 Независимые величины

Величины x и y называют независимыми если результат измерения одной
из них никак не влияет на результат измерения другой. Для таких величин вероятность того, что x окажется в некоторой области X, и одновременно y — в области Y,
равна произведению соответствующих вероятностей:

Обозначим отклонения величин от их средних как Δ⁢x=x-x¯ и
Δ⁢y=y-y¯.
Средние значения этих отклонений равны, очевидно, нулю: Δ⁢x¯=x¯-x¯=0,
Δ⁢y¯=0. Из независимости величин x и y следует,
что среднее значение от произведения Δ⁢x⋅Δ⁢y¯
равно произведению средних Δ⁢x¯⋅Δ⁢y¯
и, следовательно, равно нулю:

Δ⁢x⋅Δ⁢y¯=Δ⁢x¯⋅Δ⁢y¯=0. (2.6)

Пусть измеряемая величина z=x+y складывается из двух независимых
случайных слагаемых x и y, для которых известны средние
x¯ и y¯, и их среднеквадратичные погрешности
σx и σy. Непосредственно из определения (1.1)
следует, что среднее суммы равно сумме средних:

Найдём дисперсию σz2. В силу независимости имеем

Δ⁢z2¯=Δ⁢x2¯+Δ⁢y2¯+2⁢Δ⁢x⋅Δ⁢y¯≈Δ⁢x2¯+Δ⁢y2¯,

то есть:

Таким образом, при сложении независимых величин их погрешности
складываются среднеквадратичным образом.

Подчеркнём, что для справедливости соотношения (2.7)
величины x и y не обязаны быть нормально распределёнными —
достаточно существования конечных значений их дисперсий. Однако можно
показать, что если x и y распределены нормально, нормальным
будет и распределение их суммы
.

Замечание. Требование независимости
слагаемых является принципиальным. Например, положим y=x. Тогда
z=2⁢x. Здесь y и x, очевидно, зависят друг от друга. Используя
(2.7), находим σ2⁢x=2⁢σx,
что, конечно, неверно — непосредственно из определения
следует, что σ2⁢x=2⁢σx.

Отдельно стоит обсудить математическую структуру формулы (2.7).
Если одна из погрешностей много больше другой, например,
σx≫σy,
то меньшей погрешностью можно пренебречь, σx+y≈σx.
С другой стороны, если два источника погрешностей имеют один порядок
σx∼σy, то и σx+y∼σx∼σy.

Эти обстоятельства важны при планирования эксперимента: как правило,
величина, измеренная наименее точно, вносит наибольший вклад в погрешность
конечного результата. При этом, пока не устранены наиболее существенные
ошибки, бессмысленно гнаться за повышением точности измерения остальных
величин.

Пример. Пусть σy=σx/3,
тогда σz=σx⁢1+19≈1,05⁢σx,
то есть при различии двух погрешностей более, чем в 3 раза, поправка
к погрешности составляет менее 5%, и уже нет особого смысла в учёте
меньшей погрешности: σz≈σx. Это утверждение
касается сложения любых независимых источников погрешностей в эксперименте.

2.4 Погрешность среднего

Выборочное среднее арифметическое значение ⟨x⟩, найденное
по результатам n измерений, само является случайной величиной.
Действительно, если поставить серию одинаковых опытов по n измерений,
то в каждом опыте получится своё среднее значение, отличающееся от
предельного среднего x¯.

Вычислим среднеквадратичную погрешность среднего арифметического
σ⟨x⟩.
Рассмотрим вспомогательную сумму n слагаемых

Если {xi} есть набор независимых измерений
одной и той же физической величины, то мы можем, применяя результат
(2.7) предыдущего параграфа, записать

σZ=σx12+σx22+…+σxn2=n⁢σx,

поскольку под корнем находится n одинаковых слагаемых. Отсюда с
учётом ⟨x⟩=Z/n получаем

Таким образом, погрешность среднего значения x по результатам
n независимых измерений оказывается в n раз меньше погрешности
отдельного измерения
. Это один из важнейших результатов, позволяющий
уменьшать случайные погрешности эксперимента за счёт многократного
повторения измерений.

Подчеркнём отличия между σx и σ⟨x⟩:

величина σx — погрешность отдельного
измерения
— является характеристикой разброса значений
в совокупности измерений {xi}, i=1..n. При
нормальном законе распределения примерно 68% измерений попадают в
интервал ⟨x⟩±σx;

величина σ⟨x⟩ — погрешность
среднего
— характеризует точность, с которой определено
среднее значение измеряемой физической величины ⟨x⟩ относительно
предельного («истинного») среднего x¯;
при этом с доверительной вероятностью P=68% искомая величина x¯
лежит в интервале
⟨x⟩-σ⟨x⟩<x¯<⟨x⟩+σ⟨x⟩.

2.5 Результирующая погрешность опыта

Пусть для некоторого результата измерения известна оценка его максимальной
систематической погрешности Δсист и случайная
среднеквадратичная
погрешность σслуч. Какова «полная»
погрешность измерения?

Предположим для простоты, что измеряемая величина в принципе
может быть определена сколь угодно точно, так что можно говорить о
некотором её «истинном» значении xист
(иными словами, погрешность результата связана в основном именно с
процессом измерения). Назовём полной погрешностью измерения
среднеквадратичное значения отклонения от результата измерения от
«истинного»:

Отклонение x-xист можно представить как сумму случайного
отклонения от среднего δ⁢xслуч=x-x¯
и постоянной (но, вообще говоря, неизвестной) систематической составляющей
δ⁢xсист=x¯-xист=const:

Причём случайную составляющую можно считать независимой от систематической.
В таком случае из (2.7) находим:

σполн2=⟨δ⁢xсист2⟩+⟨δ⁢xслуч2⟩≤Δсист2+σслуч2. (2.9)

Таким образом, для получения максимального значения полной
погрешности некоторого измерения нужно квадратично сложить максимальную
систематическую и случайную погрешности.

Если измерения проводятся многократно, то согласно (2.8)
случайная составляющая погрешности может быть уменьшена, а систематическая
составляющая при этом остаётся неизменной:

Отсюда следует важное практическое правило
(см. также обсуждение в п. 2.3): если случайная погрешность измерений
в 2–3 раза меньше предполагаемой систематической, то
нет смысла проводить многократные измерения в попытке уменьшить погрешность
всего эксперимента. В такой ситуации измерения достаточно повторить
2–3 раза — чтобы убедиться в повторяемости результата, исключить промахи
и проверить, что случайная ошибка действительно мала.
В противном случае повторение измерений может иметь смысл до
тех пор, пока погрешность среднего
σ⟨x⟩=σxn
не станет меньше систематической.


Замечание. Поскольку конкретная
величина систематической погрешности, как правило, не известна, её
можно в некотором смысле рассматривать наравне со случайной —
предположить, что её величина была определена по некоторому случайному
закону перед началом измерений (например, при изготовлении линейки
на заводе произошло некоторое случайное искажение шкалы). При такой
трактовке формулу (2.9) можно рассматривать просто
как частный случай формулы сложения погрешностей независимых величин
(2.7).

Подчеркнем, что вероятностный закон, которому подчиняется
систематическая ошибка, зачастую неизвестен. Поэтому неизвестно и
распределение итогового результата. Из этого, в частности, следует,
что мы не можем приписать интервалу x±Δсист какую-либо
определённую доверительную вероятность — она равна 0,68
только если систематическая ошибка имеет нормальное распределение.
Можно, конечно, предположить,
— и так часто делают — что, к примеру, ошибки
при изготовлении линеек на заводе имеют гауссов характер. Также часто
предполагают, что систематическая ошибка имеет равномерное
распределение (то есть «истинное» значение может с равной вероятностью
принять любое значение в пределах интервала ±Δсист).
Строго говоря, для этих предположений нет достаточных оснований.


Пример. В результате измерения диаметра проволоки микрометрическим винтом,
имеющим цену деления h=0,01 мм, получен следующий набор из n=8 значений:

Вычисляем среднее значение: ⟨d⟩≈386,3 мкм.
Среднеквадратичное отклонение:
σd≈9,2 мкм. Случайная погрешность среднего согласно
(2.8):
σ⟨d⟩=σd8≈3,2
мкм. Все результаты лежат в пределах ±2⁢σd, поэтому нет
причин сомневаться в нормальности распределения. Максимальную погрешность
микрометра оценим как половину цены деления, Δ=h2=5 мкм.
Результирующая полная погрешность
σ≤Δ2+σd28≈6,0 мкм.
Видно, что σслуч≈Δсист и проводить дополнительные измерения
особого смысла нет. Окончательно результат измерений может быть представлен
в виде (см. также правила округления
результатов измерений в п. 4.3.2)



d=386±6⁢мкм,εd=1,5%.

Заметим, что поскольку случайная погрешность и погрешность
прибора здесь имеют один порядок величины, наблюдаемый случайный разброс
данных может быть связан как с неоднородностью сечения проволоки,
так и с дефектами микрометра (например, с неровностями зажимов, люфтом
винта, сухим трением, деформацией проволоки под действием микрометра
и т. п.). Для ответа на вопрос, что именно вызвало разброс, требуются
дополнительные исследования, желательно с использованием более точных
приборов.


Пример. Измерение скорости
полёта пули было осуществлено с погрешностью δ⁢v=±1 м/c.
Результаты измерений для n=6 выстрелов представлены в таблице:

Усреднённый результат ⟨v⟩=162,0⁢м/с,
среднеквадратичное отклонение σv=13,8⁢м/c, случайная
ошибка для средней скорости
σv¯=σv/6=5,6⁢м/с.
Поскольку разброс экспериментальных данных существенно превышает погрешность
каждого измерения, σv≫δ⁢v, он почти наверняка связан
с реальным различием скоростей пули в разных выстрелах, а не с ошибками
измерений. В качестве результата эксперимента представляют интерес
как среднее значение скоростей ⟨v⟩=162±6⁢м/с
(ε≈4%), так и значение σv≈14⁢м/с,
характеризующее разброс значений скоростей от выстрела к выстрелу.
Малая инструментальная погрешность в принципе позволяет более точно
измерить среднее и дисперсию, и исследовать закон распределения выстрелов
по скоростям более детально — для этого требуется набрать
бо́льшую статистику по выстрелам.


Пример. Измерение скорости
полёта пули было осуществлено с погрешностью δ⁢v=10 м/c. Результаты
измерений для n=6 выстрелов представлены в таблице:

Усреднённый результат ⟨v⟩=163,3⁢м/с,
σv=12,1⁢м/c, σ⟨v⟩=5⁢м/с,
σполн≈11,2⁢м/с. Инструментальная
погрешность каждого измерения превышает разброс данных, поэтому в
этом опыте затруднительно сделать вывод о различии скоростей от выстрела
к выстрелу. Результат измерений скорости пули:
⟨v⟩=163±11⁢м/с,
ε≈7%. Проводить дополнительные выстрелы при такой
большой инструментальной погрешности особого смысла нет —
лучше поработать над точностью приборов и методикой измерений.

2.6 Обработка косвенных измерений

Косвенными называют измерения, полученные в результате расчётов,
использующих результаты прямых (то есть «непосредственных»)
измерений физических величин. Сформулируем основные правила пересчёта
погрешностей при косвенных измерениях.

2.6.1 Случай одной переменной

Пусть в эксперименте измеряется величина x, а её «наилучшее»
(в некотором смысле) значение равно x⋆ и оно известно с
погрешностью σx. После чего с помощью известной функции
вычисляется величина y=f⁢(x).

В качестве «наилучшего» приближения для y используем значение функции
при «наилучшем» x:

Найдём величину погрешности σy. Обозначая отклонение измеряемой
величины как Δ⁢x=x-x⋆, и пользуясь определением производной,
при условии, что функция y⁢(x) — гладкая
вблизи x≈x⋆, запишем

где f′≡d⁢yd⁢x — производная фукнции f⁢(x), взятая в точке
x⋆. Возведём полученное в квадрат, проведём усреднение
(σy2=⟨Δ⁢y2⟩,
σx2=⟨Δ⁢x2⟩), и затем снова извлечём
корень. В результате получим


Пример. Для степенной функции
y=A⁢xn имеем σy=n⁢A⁢xn-1⁢σx, откуда



σyy=n⁢σxx,или  εy=n⁢εx,

то есть относительная погрешность степенной функции возрастает пропорционально
показателю степени n.

Пример. Для y=1/x имеем ε1/x=εx
— при обращении величины сохраняется её относительная
погрешность.

Упражнение. Найдите погрешность логарифма y=ln⁡x, если известны x
и σx.

Упражнение. Найдите погрешность показательной функции y=ax,
если известны x и σx. Коэффициент a задан точно.

2.6.2 Случай многих переменных

Пусть величина u вычисляется по измеренным значениям нескольких
различных независимых физических величин x, y, …
на основе известного закона u=f⁢(x,y,…). В качестве
наилучшего значения можно по-прежнему взять значение функции f
при наилучших значениях измеряемых параметров:

Для нахождения погрешности σu воспользуемся свойством,
известным из математического анализа, — малые приращения гладких
функции многих переменных складываются линейно, то есть справедлив
принцип суперпозиции малых приращений:

где символом fx′≡∂⁡f∂⁡x обозначена
частная производная функции f по переменной x —
то есть обычная производная f по x, взятая при условии, что
все остальные аргументы (кроме x) считаются постоянными параметрами.
Тогда пользуясь формулой для нахождения дисперсии суммы независимых
величин (2.7), получим соотношение, позволяющее вычислять
погрешности косвенных измерений для произвольной функции
u=f⁢(x,y,…):

σu2=fx′⁣2⁢σx2+fy′⁣2⁢σy2+… (2.11)

Это и есть искомая общая формула пересчёта погрешностей при косвенных
измерениях.

Отметим, что формулы (2.10) и (2.11) применимы
только если относительные отклонения всех величин малы
(εx,εy,…≪1),
а измерения проводятся вдали от особых точек функции f (производные
fx′, fy′ … не должны обращаться в бесконечность).
Также подчеркнём, что все полученные здесь формулы справедливы только
для независимых переменных x, y, …

Остановимся на некоторых важных частных случаях формулы
(2.11).


Пример. Для суммы (или разности) u=∑i=1nai⁢xi имеем



σu2=∑i=1nai2⁢σxi2.

(2.12)


Пример. Найдём погрешность степенной функции:
u=xα⋅yβ⋅…. Тогда нетрудно получить,
что



σu2u2=α2⁢σx2x2+β2⁢σy2y2+…

или через относительные погрешности



εu2=α2⁢εx2+β2⁢εy2+…

(2.13)


Пример. Вычислим погрешность произведения и частного: u=x⁢y или u=x/y.
Тогда в обоих случаях имеем



εu2=εx2+εy2,

(2.14)

то есть при умножении или делении относительные погрешности складываются
квадратично.


Пример. Рассмотрим несколько более сложный случай: нахождение угла по его тангенсу



u=arctgyx.

В таком случае, пользуясь тем, что (arctgz)′=11+z2,
где z=y/x, и используя производную сложной функции, находим
ux′=uz′⁢zx′=-yx2+y2,
uy′=uz′⁢zy′=xx2+y2, и наконец



σu2=y2⁢σx2+x2⁢σy2(x2+y2)2.

Упражнение. Найти погрешность вычисления гипотенузы z=x2+y2
прямоугольного треугольника по измеренным катетам x и y.

По итогам данного раздела можно дать следующие практические рекомендации.

  • Как правило, нет смысла увеличивать точность измерения какой-то одной
    величины, если другие величины, используемые в расчётах, остаются
    измеренными относительно грубо — всё равно итоговая погрешность
    скорее всего будет определяться самым неточным измерением. Поэтому
    все измерения имеет смысл проводить примерно с одной и той же
    относительной погрешностью
    .

  • При этом, как следует из (2.13), особое внимание
    следует уделять измерению величин, возводимых при расчётах в степени
    с большими показателями. А при сложных функциональных зависимостях
    имеет смысл детально проанализировать структуру формулы
    (2.11):
    если вклад от некоторой величины в общую погрешность мал, нет смысла
    гнаться за высокой точностью её измерения, и наоборот, точность некоторых
    измерений может оказаться критически важной.

  • Следует избегать измерения малых величин как разности двух близких
    значений (например, толщины стенки цилиндра как разности внутреннего
    и внешнего радиусов): если u=x-y, то абсолютная погрешность
    σu=σx2+σy2
    меняется мало, однако относительная погрешность
    εu=σux-y
    может оказаться неприемлемо большой, если x≈y.

Результат любого измерения не определён однозначно и имеет случайную составляющую.
Поэтому адекватным языком для описания погрешностей является язык вероятностей.
Тот факт, что значение некоторой величины «случайно», не означает, что
она может принимать совершенно произвольные значения. Ясно, что частоты, с которыми
возникает те или иные значения, различны. Вероятностные законы, которым
подчиняются случайные величины, называют распределениями.

2.1 Случайная величина

Случайной будем называть величину, значение которой не может быть достоверно определено экспериментатором. Чаще всего подразумевается, что случайная величина будет изменяться при многократном повторении одного и того же эксперимента. При интерпретации результатов измерений в физических экспериментах, обычно случайными также считаются величины, значение которых является фиксированным, но не известно экспериментатору. Например смещение нуля шкалы прибора. Для формализации работы со случайными величинами используют понятие вероятности. Численное значение вероятности того, что какая-то величина примет то или иное значение определяется либо как относительная частота наблюдения того или иного значения при повторении опыта большое количество раз, либо как оценка на основе данных других экспериментов.

Замечание. 
Хотя понятия вероятности и случайной величины являются основополагающими, в литературе нет единства в их определении. Обсуждение формальных тонкостей или построение строгой теории лежит за пределами данного пособия. Поэтому на начальном этапе лучше использовать «интуитивное» понимание этих сущностей. Заинтересованным читателям рекомендуем обратиться к специальной литературе: [5].

Рассмотрим случайную физическую величину x, которая при измерениях может
принимать непрерывный набор значений. Пусть
P[x0,x0+δ⁢x] — вероятность того, что результат окажется вблизи
некоторой точки x0 в пределах интервала δ⁢x: x∈[x0,x0+δ⁢x].
Устремим интервал
δ⁢x к нулю. Нетрудно понять, что вероятность попасть в этот интервал
также будет стремиться к нулю. Однако отношение
w⁢(x0)=P[x0,x0+δ⁢x]δ⁢x будет оставаться конечным.
Функцию w⁢(x) называют плотностью распределения вероятности или кратко
распределением непрерывной случайной величины x.

Замечание. В математической литературе распределением часто называют не функцию
w⁢(x), а её интеграл W⁢(x)=∫w⁢(x)⁢𝑑x. Такую функцию в физике принято
называть интегральным или кумулятивным распределением. В англоязычной литературе
для этих функций принято использовать сокращения:
pdf (probability distribution function) и
cdf (cumulative distribution function)
соответственно.

Гистограммы.

Проиллюстрируем наглядно понятие плотности распределения. Результат
большого числа измерений случайной величины удобно представить с помощью
специального типа графика — гистограммы.
Для этого область значений x, размещённую на оси абсцисс, разобьём на
равные малые интервалы — «корзины» или «бины» (англ. bins)
некоторого размера h. По оси ординат будем откладывать долю измерений w,
результаты которых попадают в соответствующую корзину. А именно,
пусть k — номер корзины; nk — число измерений, попавших
в диапазон x∈[k⁢h,(k+1)⁢h]. Тогда на графике изобразим «столбик»
шириной h и высотой wk=nk/n.
В результате получим картину, подобную изображённой на рис. 2.1.

Рис. 2.1: Пример гистограммы для нормального распределения (x¯=10,
σ=1,0, h=0,1, n=104)

Высоты построенных столбиков будут приближённо соответствовать значению
плотности распределения w⁢(x) вблизи соответствующей точки x.
Если устремить число измерений к бесконечности (n→∞), а ширину корзин
к нулю (h→0), то огибающая гистограммы будет стремиться к некоторой
непрерывной функции w⁢(x).

Самые высокие столбики гистограммы будут группироваться вблизи максимума
функции w⁢(x) — это наиболее вероятное значение случайной величины.
Если отклонения в положительную и отрицательную стороны равновероятны,
то гистограмма будет симметрична — в таком случае среднее значение ⟨x⟩
также будет лежать вблизи этого максимума. Ширина гистограммы будет характеризовать разброс
значений случайной величины — по порядку величины
она, как правило, близка к среднеквадратичному отклонению sx.

Свойства распределений.

Из определения функции w⁢(x) следует, что вероятность получить в результате
эксперимента величину x в диапазоне от a до b
можно найти, вычислив интеграл:

Px∈[a,b]=∫abw⁢(x)⁢𝑑x. (2.1)

Согласно определению вероятности, сумма вероятностей для всех возможных случаев
всегда равна единице. Поэтому интеграл распределения w⁢(x) по всей области
значений x (то есть суммарная площадь под графиком w⁢(x)) равен единице:

Это соотношение называют условием нормировки.

Среднее и дисперсия.

Вычислим среднее по построенной гистограмме. Если размер корзин
h достаточно мал, все измерения в пределах одной корзины можно считать примерно
одинаковыми. Тогда среднее арифметическое всех результатов можно вычислить как

Переходя к пределу, получим следующее определение среднего значения
случайной величины:

где интегрирование ведётся по всей области значений x.
В теории вероятностей x¯ также называют математическим ожиданием
распределения.
Величину

σ2=(x-x¯)2¯=∫(x-x¯)2⁢w⁢𝑑x (2.3)

называют дисперсией распределения. Значение σ есть
срекднеквадратичное отклонение в пределе n→∞. Оно имеет ту
же размерность, что и сама величина x и характеризует разброс распределения.
Именно эту величину, как правило, приводят как характеристику погрешности
измерения x.

Доверительный интервал.

Обозначим как P|Δ⁢x|<δ вероятность
того, что отклонение от среднего Δ⁢x=x-x¯ составит величину,
не превосходящую по модулю значение δ:

P|Δ⁢x|<δ=∫x¯-δx¯+δw⁢(x)⁢𝑑x. (2.4)

Эту величину называют доверительной вероятностью для
доверительного интервала |x-x¯|≤δ.

2.2 Нормальное распределение

Одним из наиболее примечательных результатов теории вероятностей является
так называемая центральная предельная теорема. Она утверждает,
что сумма большого количества независимых случайных слагаемых, каждое
из которых вносит в эту сумму относительно малый вклад, подчиняется
универсальному закону, не зависимо от того, каким вероятностным законам
подчиняются её составляющие, — так называемому нормальному
распределению
(или распределению Гаусса).

Доказательство теоремы довольно громоздко и мы его не приводим (его можно найти
в любом учебнике по теории вероятностей). Остановимся
кратко на том, что такое нормальное распределение и его основных свойствах.

Плотность нормального распределения выражается следующей формулой:

w𝒩⁢(x)=12⁢π⁢σ⁢e-(x-x¯)22⁢σ2. (2.5)

Здесь x¯ и σ
— параметры нормального распределения: x¯ равно
среднему значению x, a σ —
среднеквадратичному отклонению, вычисленным в пределе n→∞.

Как видно из рис. 2.1, распределение представляет собой
симметричный
«колокол», положение вершины которого
соответствует x¯ (ввиду симметрии оно же
совпадает с наиболее вероятным значением — максимумом
функции w𝒩⁢(x)).

При значительном отклонении x от среднего величина
w𝒩⁢(x)
очень быстро убывает. Это означает, что вероятность встретить отклонения,
существенно большие, чем σ, оказывается пренебрежимо
мала
. Ширина «колокола» по порядку величины
равна σ — она характеризует «разброс»
экспериментальных данных относительно среднего значения.

Замечание. Точки x=x¯±σ являются точками
перегиба графика w⁢(x) (в них вторая производная по x
обращается в нуль, w′′=0), а их положение по высоте составляет
w⁢(x¯±σ)/w⁢(x¯)=e-1/2≈0,61
от высоты вершины.

Универсальный характер центральной предельной теоремы позволяет широко
применять на практике нормальное (гауссово) распределение для обработки
результатов измерений, поскольку часто случайные погрешности складываются из
множества случайных независимых факторов. Заметим, что на практике
для приближённой оценки параметров нормального распределения
случайной величины используются выборочные значения среднего
и дисперсии: x¯≈⟨x⟩, sx≈σx.

x-x0σ2=2w⁢(x)σ1=1

Рис. 2.2: Плотность нормального распределения

Доверительные вероятности.

Вычислим некоторые доверительные вероятности (2.4) для нормально
распределённых случайных величин.

Замечание. Значение интеграла вида ∫e-x2/2⁢𝑑x
(его называют интегралом ошибок) в элементарных функциях не выражается,
но легко находится численно.

Вероятность того, что результат отдельного измерения x окажется
в пределах x¯±σ оказывается равна

P|Δ⁢x|<σ=∫x¯-σx¯+σw𝒩⁢𝑑x≈0,68.

Вероятность отклонения в пределах x¯±2⁢σ:

а в пределах x¯±3⁢σ:

Иными словами, при большом числе измерений нормально распределённой
величины можно ожидать, что лишь треть измерений выпадут за пределы интервала
[x¯-σ,x¯+σ]. При этом около 5%
измерений выпадут за пределы [x¯-2⁢σ;x¯+2⁢σ],
и лишь 0,27% окажутся за пределами
[x¯-3⁢σ;x¯+3⁢σ].

Пример. В сообщениях об открытии бозона Хиггса на Большом адронном коллайдере
говорилось о том, что исследователи ждали подтверждение результатов
с точностью «5 сигма». Используя нормальное распределение (2.5)
нетрудно посчитать, что они использовали доверительную вероятность
P≈1-5,7⋅10-7=0,99999943. Такую точность можно назвать фантастической.

Полученные значения доверительных вероятностей используются при
стандартной записи результатов измерений. В физических измерениях
(в частности, в учебной лаборатории), как правило, используется P=0,68,
то есть, запись

означает, что измеренное значение лежит в диапазоне (доверительном
интервале) x∈[x¯-δ⁢x;x¯+δ⁢x] с
вероятностью 68%. Таким образом погрешность ±δ⁢x считается
равной одному среднеквадратичному отклонению: δ⁢x=σ.
В технических измерениях чаще используется P=0,95, то есть под
абсолютной погрешностью имеется в виду удвоенное среднеквадратичное
отклонение, δ⁢x=2⁢σ. Во избежание разночтений доверительную
вероятность следует указывать отдельно.

Замечание. Хотя нормальный закон распределения встречается на практике довольно
часто, стоит помнить, что он реализуется далеко не всегда.
Полученные выше соотношения для вероятностей попадания значений в
доверительные интервалы можно использовать в качестве простейшего
признака нормальности распределения: в частности, если количество попадающих
в интервал ±σ результатов существенно отличается от 2/3 — это повод
для более детального исследования закона распределения ошибок.

Сравнение результатов измерений.

Теперь мы можем дать количественный критерий для сравнения двух измеренных
величин или двух результатов измерения одной и той же величины.

Пусть x1 и x2 (x1≠x2) измерены с
погрешностями σ1 и σ2 соответственно.
Ясно, что если различие результатов |x2-x1| невелико,
его можно объяснить просто случайными отклонениями.
Если же теория предсказывает, что вероятность обнаружить такое отклонение
слишком мала, различие результатов следует признать значимым.
Предварительно необходимо договориться о соответствующем граничном значении
вероятности. Универсального значения здесь быть не может,
поэтому приходится полагаться на субъективный выбор исследователя. Часто
в качестве «разумной» границы выбирают вероятность 5%,
что, как видно из изложенного выше, для нормального распределения
соответствует отклонению более, чем на 2⁢σ.

Допустим, одна из величин известна с существенно большей точностью:
σ2≪σ1 (например, x1 — результат, полученный
студентом в лаборатории, x2 — справочное значение).
Поскольку σ2 мало, x2 можно принять за «истинное»:
x2≈x¯. Предполагая, что погрешность измерения
x1 подчиняется нормальному закону с и дисперсией σ12,
можно утверждать, что
различие считают будет значимы, если

Пусть погрешности измерений сравнимы по порядку величины:
σ1∼σ2. В теории вероятностей показывается, что
линейная комбинация нормально распределённых величин также имеет нормальное
распределение с дисперсией σ2=σ12+σ22
(см. также правила сложения погрешностей (2.7)). Тогда
для проверки гипотезы о том, что x1 и x2 являются измерениями
одной и той же величины, нужно вычислить, является ли значимым отклонение
|x1-x2| от нуля при σ=σ12+σ22.


Пример. Два студента получили следующие значения для теплоты испарения
некоторой жидкости: x1=40,3±0,2 кДж/моль и
x2=41,0±0,3 кДж/моль, где погрешность соответствует
одному стандартному отклонению. Можно ли утверждать, что они исследовали
одну и ту же жидкость?

Имеем наблюдаемую разность |x1-x2|=0,7 кДж/моль,
среднеквадратичное отклонение для разности
σ=0,22+0,32=0,36 кДж/моль.
Их отношение |x2-x1|σ≈2. Из
свойств нормального распределения находим вероятность того, что измерялась
одна и та же величина, а различия в ответах возникли из-за случайных
ошибок: P≈5%. Ответ на вопрос, «достаточно»
ли мала или велика эта вероятность, остаётся на усмотрение исследователя.

Замечание. Изложенные здесь соображения применимы, только если x¯ и
его стандартное отклонение σ получены на основании достаточно
большой выборки n≫1 (или заданы точно). При небольшом числе измерений
(n≲10) выборочные средние ⟨x⟩ и среднеквадратичное отклонение
sx сами имеют довольно большую ошибку, а
их распределение будет описываться не нормальным законом, а так
называемым t-распределением Стъюдента. В частности, в зависимости от
значения n интервал ⟨x⟩±sx будет соответствовать несколько
меньшей доверительной вероятности, чем P=0,68. Особенно резко различия
проявляются при высоких уровнях доверительных вероятностей P→1.

2.3 Независимые величины

Величины x и y называют независимыми если результат измерения одной
из них никак не влияет на результат измерения другой. Для таких величин вероятность того, что x окажется в некоторой области X, и одновременно y — в области Y,
равна произведению соответствующих вероятностей:

Обозначим отклонения величин от их средних как Δ⁢x=x-x¯ и
Δ⁢y=y-y¯.
Средние значения этих отклонений равны, очевидно, нулю: Δ⁢x¯=x¯-x¯=0,
Δ⁢y¯=0. Из независимости величин x и y следует,
что среднее значение от произведения Δ⁢x⋅Δ⁢y¯
равно произведению средних Δ⁢x¯⋅Δ⁢y¯
и, следовательно, равно нулю:

Δ⁢x⋅Δ⁢y¯=Δ⁢x¯⋅Δ⁢y¯=0. (2.6)

Пусть измеряемая величина z=x+y складывается из двух независимых
случайных слагаемых x и y, для которых известны средние
x¯ и y¯, и их среднеквадратичные погрешности
σx и σy. Непосредственно из определения (1.1)
следует, что среднее суммы равно сумме средних:

Найдём дисперсию σz2. В силу независимости имеем

Δ⁢z2¯=Δ⁢x2¯+Δ⁢y2¯+2⁢Δ⁢x⋅Δ⁢y¯≈Δ⁢x2¯+Δ⁢y2¯,

то есть:

Таким образом, при сложении независимых величин их погрешности
складываются среднеквадратичным образом.

Подчеркнём, что для справедливости соотношения (2.7)
величины x и y не обязаны быть нормально распределёнными —
достаточно существования конечных значений их дисперсий. Однако можно
показать, что если x и y распределены нормально, нормальным
будет и распределение их суммы
.

Замечание. Требование независимости
слагаемых является принципиальным. Например, положим y=x. Тогда
z=2⁢x. Здесь y и x, очевидно, зависят друг от друга. Используя
(2.7), находим σ2⁢x=2⁢σx,
что, конечно, неверно — непосредственно из определения
следует, что σ2⁢x=2⁢σx.

Отдельно стоит обсудить математическую структуру формулы (2.7).
Если одна из погрешностей много больше другой, например,
σx≫σy,
то меньшей погрешностью можно пренебречь, σx+y≈σx.
С другой стороны, если два источника погрешностей имеют один порядок
σx∼σy, то и σx+y∼σx∼σy.

Эти обстоятельства важны при планирования эксперимента: как правило,
величина, измеренная наименее точно, вносит наибольший вклад в погрешность
конечного результата. При этом, пока не устранены наиболее существенные
ошибки, бессмысленно гнаться за повышением точности измерения остальных
величин.

Пример. Пусть σy=σx/3,
тогда σz=σx⁢1+19≈1,05⁢σx,
то есть при различии двух погрешностей более, чем в 3 раза, поправка
к погрешности составляет менее 5%, и уже нет особого смысла в учёте
меньшей погрешности: σz≈σx. Это утверждение
касается сложения любых независимых источников погрешностей в эксперименте.

2.4 Погрешность среднего

Выборочное среднее арифметическое значение ⟨x⟩, найденное
по результатам n измерений, само является случайной величиной.
Действительно, если поставить серию одинаковых опытов по n измерений,
то в каждом опыте получится своё среднее значение, отличающееся от
предельного среднего x¯.

Вычислим среднеквадратичную погрешность среднего арифметического
σ⟨x⟩.
Рассмотрим вспомогательную сумму n слагаемых

Если {xi} есть набор независимых измерений
одной и той же физической величины, то мы можем, применяя результат
(2.7) предыдущего параграфа, записать

σZ=σx12+σx22+…+σxn2=n⁢σx,

поскольку под корнем находится n одинаковых слагаемых. Отсюда с
учётом ⟨x⟩=Z/n получаем

Таким образом, погрешность среднего значения x по результатам
n независимых измерений оказывается в n раз меньше погрешности
отдельного измерения
. Это один из важнейших результатов, позволяющий
уменьшать случайные погрешности эксперимента за счёт многократного
повторения измерений.

Подчеркнём отличия между σx и σ⟨x⟩:

величина σx — погрешность отдельного
измерения
— является характеристикой разброса значений
в совокупности измерений {xi}, i=1..n. При
нормальном законе распределения примерно 68% измерений попадают в
интервал ⟨x⟩±σx;

величина σ⟨x⟩ — погрешность
среднего
— характеризует точность, с которой определено
среднее значение измеряемой физической величины ⟨x⟩ относительно
предельного («истинного») среднего x¯;
при этом с доверительной вероятностью P=68% искомая величина x¯
лежит в интервале
⟨x⟩-σ⟨x⟩<x¯<⟨x⟩+σ⟨x⟩.

2.5 Результирующая погрешность опыта

Пусть для некоторого результата измерения известна оценка его максимальной
систематической погрешности Δсист и случайная
среднеквадратичная
погрешность σслуч. Какова «полная»
погрешность измерения?

Предположим для простоты, что измеряемая величина в принципе
может быть определена сколь угодно точно, так что можно говорить о
некотором её «истинном» значении xист
(иными словами, погрешность результата связана в основном именно с
процессом измерения). Назовём полной погрешностью измерения
среднеквадратичное значения отклонения от результата измерения от
«истинного»:

Отклонение x-xист можно представить как сумму случайного
отклонения от среднего δ⁢xслуч=x-x¯
и постоянной (но, вообще говоря, неизвестной) систематической составляющей
δ⁢xсист=x¯-xист=const:

Причём случайную составляющую можно считать независимой от систематической.
В таком случае из (2.7) находим:

σполн2=⟨δ⁢xсист2⟩+⟨δ⁢xслуч2⟩≤Δсист2+σслуч2. (2.9)

Таким образом, для получения максимального значения полной
погрешности некоторого измерения нужно квадратично сложить максимальную
систематическую и случайную погрешности.

Если измерения проводятся многократно, то согласно (2.8)
случайная составляющая погрешности может быть уменьшена, а систематическая
составляющая при этом остаётся неизменной:

Отсюда следует важное практическое правило
(см. также обсуждение в п. 2.3): если случайная погрешность измерений
в 2–3 раза меньше предполагаемой систематической, то
нет смысла проводить многократные измерения в попытке уменьшить погрешность
всего эксперимента. В такой ситуации измерения достаточно повторить
2–3 раза — чтобы убедиться в повторяемости результата, исключить промахи
и проверить, что случайная ошибка действительно мала.
В противном случае повторение измерений может иметь смысл до
тех пор, пока погрешность среднего
σ⟨x⟩=σxn
не станет меньше систематической.


Замечание. Поскольку конкретная
величина систематической погрешности, как правило, не известна, её
можно в некотором смысле рассматривать наравне со случайной —
предположить, что её величина была определена по некоторому случайному
закону перед началом измерений (например, при изготовлении линейки
на заводе произошло некоторое случайное искажение шкалы). При такой
трактовке формулу (2.9) можно рассматривать просто
как частный случай формулы сложения погрешностей независимых величин
(2.7).

Подчеркнем, что вероятностный закон, которому подчиняется
систематическая ошибка, зачастую неизвестен. Поэтому неизвестно и
распределение итогового результата. Из этого, в частности, следует,
что мы не можем приписать интервалу x±Δсист какую-либо
определённую доверительную вероятность — она равна 0,68
только если систематическая ошибка имеет нормальное распределение.
Можно, конечно, предположить,
— и так часто делают — что, к примеру, ошибки
при изготовлении линеек на заводе имеют гауссов характер. Также часто
предполагают, что систематическая ошибка имеет равномерное
распределение (то есть «истинное» значение может с равной вероятностью
принять любое значение в пределах интервала ±Δсист).
Строго говоря, для этих предположений нет достаточных оснований.


Пример. В результате измерения диаметра проволоки микрометрическим винтом,
имеющим цену деления h=0,01 мм, получен следующий набор из n=8 значений:

Вычисляем среднее значение: ⟨d⟩≈386,3 мкм.
Среднеквадратичное отклонение:
σd≈9,2 мкм. Случайная погрешность среднего согласно
(2.8):
σ⟨d⟩=σd8≈3,2
мкм. Все результаты лежат в пределах ±2⁢σd, поэтому нет
причин сомневаться в нормальности распределения. Максимальную погрешность
микрометра оценим как половину цены деления, Δ=h2=5 мкм.
Результирующая полная погрешность
σ≤Δ2+σd28≈6,0 мкм.
Видно, что σслуч≈Δсист и проводить дополнительные измерения
особого смысла нет. Окончательно результат измерений может быть представлен
в виде (см. также правила округления
результатов измерений в п. 4.3.2)



d=386±6⁢мкм,εd=1,5%.


Заметим, что поскольку случайная погрешность и погрешность
прибора здесь имеют один порядок величины, наблюдаемый случайный разброс
данных может быть связан как с неоднородностью сечения проволоки,
так и с дефектами микрометра (например, с неровностями зажимов, люфтом
винта, сухим трением, деформацией проволоки под действием микрометра
и т. п.). Для ответа на вопрос, что именно вызвало разброс, требуются
дополнительные исследования, желательно с использованием более точных
приборов.


Пример. Измерение скорости
полёта пули было осуществлено с погрешностью δ⁢v=±1 м/c.
Результаты измерений для n=6 выстрелов представлены в таблице:

Усреднённый результат ⟨v⟩=162,0⁢м/с,
среднеквадратичное отклонение σv=13,8⁢м/c, случайная
ошибка для средней скорости
σv¯=σv/6=5,6⁢м/с.
Поскольку разброс экспериментальных данных существенно превышает погрешность
каждого измерения, σv≫δ⁢v, он почти наверняка связан
с реальным различием скоростей пули в разных выстрелах, а не с ошибками
измерений. В качестве результата эксперимента представляют интерес
как среднее значение скоростей ⟨v⟩=162±6⁢м/с
(ε≈4%), так и значение σv≈14⁢м/с,
характеризующее разброс значений скоростей от выстрела к выстрелу.
Малая инструментальная погрешность в принципе позволяет более точно
измерить среднее и дисперсию, и исследовать закон распределения выстрелов
по скоростям более детально — для этого требуется набрать
бо́льшую статистику по выстрелам.


Пример. Измерение скорости
полёта пули было осуществлено с погрешностью δ⁢v=10 м/c. Результаты
измерений для n=6 выстрелов представлены в таблице:

Усреднённый результат ⟨v⟩=163,3⁢м/с,
σv=12,1⁢м/c, σ⟨v⟩=5⁢м/с,
σполн≈11,2⁢м/с. Инструментальная
погрешность каждого измерения превышает разброс данных, поэтому в
этом опыте затруднительно сделать вывод о различии скоростей от выстрела
к выстрелу. Результат измерений скорости пули:
⟨v⟩=163±11⁢м/с,
ε≈7%. Проводить дополнительные выстрелы при такой
большой инструментальной погрешности особого смысла нет —
лучше поработать над точностью приборов и методикой измерений.

2.6 Обработка косвенных измерений

Косвенными называют измерения, полученные в результате расчётов,
использующих результаты прямых (то есть «непосредственных»)
измерений физических величин. Сформулируем основные правила пересчёта
погрешностей при косвенных измерениях.

2.6.1 Случай одной переменной

Пусть в эксперименте измеряется величина x, а её «наилучшее»
(в некотором смысле) значение равно x⋆ и оно известно с
погрешностью σx. После чего с помощью известной функции
вычисляется величина y=f⁢(x).

В качестве «наилучшего» приближения для y используем значение функции
при «наилучшем» x:

Найдём величину погрешности σy. Обозначая отклонение измеряемой
величины как Δ⁢x=x-x⋆, и пользуясь определением производной,
при условии, что функция y⁢(x) — гладкая
вблизи x≈x⋆, запишем

где f′≡d⁢yd⁢x — производная фукнции f⁢(x), взятая в точке
x⋆. Возведём полученное в квадрат, проведём усреднение
(σy2=⟨Δ⁢y2⟩,
σx2=⟨Δ⁢x2⟩), и затем снова извлечём
корень. В результате получим


Пример. Для степенной функции
y=A⁢xn имеем σy=n⁢A⁢xn-1⁢σx, откуда



σyy=n⁢σxx,или  εy=n⁢εx,


то есть относительная погрешность степенной функции возрастает пропорционально
показателю степени n.

Пример. Для y=1/x имеем ε1/x=εx
— при обращении величины сохраняется её относительная
погрешность.

Упражнение. Найдите погрешность логарифма y=ln⁡x, если известны x
и σx.

Упражнение. Найдите погрешность показательной функции y=ax,
если известны x и σx. Коэффициент a задан точно.

2.6.2 Случай многих переменных

Пусть величина u вычисляется по измеренным значениям нескольких
различных независимых физических величин x, y, …
на основе известного закона u=f⁢(x,y,…). В качестве
наилучшего значения можно по-прежнему взять значение функции f
при наилучших значениях измеряемых параметров:

Для нахождения погрешности σu воспользуемся свойством,
известным из математического анализа, — малые приращения гладких
функции многих переменных складываются линейно, то есть справедлив
принцип суперпозиции малых приращений:

где символом fx′≡∂⁡f∂⁡x обозначена
частная производная функции f по переменной x —
то есть обычная производная f по x, взятая при условии, что
все остальные аргументы (кроме x) считаются постоянными параметрами.
Тогда пользуясь формулой для нахождения дисперсии суммы независимых
величин (2.7), получим соотношение, позволяющее вычислять
погрешности косвенных измерений для произвольной функции
u=f⁢(x,y,…):

σu2=fx′⁣2⁢σx2+fy′⁣2⁢σy2+… (2.11)

Это и есть искомая общая формула пересчёта погрешностей при косвенных
измерениях.

Отметим, что формулы (2.10) и (2.11) применимы
только если относительные отклонения всех величин малы
(εx,εy,…≪1),
а измерения проводятся вдали от особых точек функции f (производные
fx′, fy′ … не должны обращаться в бесконечность).
Также подчеркнём, что все полученные здесь формулы справедливы только
для независимых переменных x, y, …

Остановимся на некоторых важных частных случаях формулы
(2.11).


Пример. Для суммы (или разности) u=∑i=1nai⁢xi имеем



σu2=∑i=1nai2⁢σxi2.

(2.12)



Пример. Найдём погрешность степенной функции:
u=xα⋅yβ⋅…. Тогда нетрудно получить,
что



σu2u2=α2⁢σx2x2+β2⁢σy2y2+…


или через относительные погрешности



εu2=α2⁢εx2+β2⁢εy2+…

(2.13)



Пример. Вычислим погрешность произведения и частного: u=x⁢y или u=x/y.
Тогда в обоих случаях имеем



εu2=εx2+εy2,

(2.14)


то есть при умножении или делении относительные погрешности складываются
квадратично.


Пример. Рассмотрим несколько более сложный случай: нахождение угла по его тангенсу



u=arctgyx.


В таком случае, пользуясь тем, что (arctgz)′=11+z2,
где z=y/x, и используя производную сложной функции, находим
ux′=uz′⁢zx′=-yx2+y2,
uy′=uz′⁢zy′=xx2+y2, и наконец



σu2=y2⁢σx2+x2⁢σy2(x2+y2)2.


Упражнение. Найти погрешность вычисления гипотенузы z=x2+y2
прямоугольного треугольника по измеренным катетам x и y.

По итогам данного раздела можно дать следующие практические рекомендации.

  • Как правило, нет смысла увеличивать точность измерения какой-то одной
    величины, если другие величины, используемые в расчётах, остаются
    измеренными относительно грубо — всё равно итоговая погрешность
    скорее всего будет определяться самым неточным измерением. Поэтому
    все измерения имеет смысл проводить примерно с одной и той же
    относительной погрешностью
    .

  • При этом, как следует из (2.13), особое внимание
    следует уделять измерению величин, возводимых при расчётах в степени
    с большими показателями. А при сложных функциональных зависимостях
    имеет смысл детально проанализировать структуру формулы
    (2.11):
    если вклад от некоторой величины в общую погрешность мал, нет смысла
    гнаться за высокой точностью её измерения, и наоборот, точность некоторых
    измерений может оказаться критически важной.

  • Следует избегать измерения малых величин как разности двух близких
    значений (например, толщины стенки цилиндра как разности внутреннего
    и внешнего радиусов): если u=x-y, то абсолютная погрешность
    σu=σx2+σy2
    меняется мало, однако относительная погрешность
    εu=σux-y
    может оказаться неприемлемо большой, если x≈y.

Результат любого измерения не определён однозначно и имеет случайную составляющую.
Поэтому адекватным языком для описания погрешностей является язык вероятностей.
Тот факт, что значение некоторой величины «случайно», не означает, что
она может принимать совершенно произвольные значения. Ясно, что частоты, с которыми
возникает те или иные значения, различны. Вероятностные законы, которым
подчиняются случайные величины, называют распределениями.

2.1 Случайная величина

Случайной будем называть величину, значение которой не может быть достоверно определено экспериментатором. Чаще всего подразумевается, что случайная величина будет изменяться при многократном повторении одного и того же эксперимента. При интерпретации результатов измерений в физических экспериментах, обычно случайными также считаются величины, значение которых является фиксированным, но не известно экспериментатору. Например смещение нуля шкалы прибора. Для формализации работы со случайными величинами используют понятие вероятности. Численное значение вероятности того, что какая-то величина примет то или иное значение определяется либо как относительная частота наблюдения того или иного значения при повторении опыта большое количество раз, либо как оценка на основе данных других экспериментов.

Замечание. 
Хотя понятия вероятности и случайной величины являются основополагающими, в литературе нет единства в их определении. Обсуждение формальных тонкостей или построение строгой теории лежит за пределами данного пособия. Поэтому на начальном этапе лучше использовать «интуитивное» понимание этих сущностей. Заинтересованным читателям рекомендуем обратиться к специальной литературе: [5].

Рассмотрим случайную физическую величину x, которая при измерениях может
принимать непрерывный набор значений. Пусть
P[x0,x0+δ⁢x] — вероятность того, что результат окажется вблизи
некоторой точки x0 в пределах интервала δ⁢x: x∈[x0,x0+δ⁢x].
Устремим интервал
δ⁢x к нулю. Нетрудно понять, что вероятность попасть в этот интервал
также будет стремиться к нулю. Однако отношение
w⁢(x0)=P[x0,x0+δ⁢x]δ⁢x будет оставаться конечным.
Функцию w⁢(x) называют плотностью распределения вероятности или кратко
распределением непрерывной случайной величины x.

Замечание. В математической литературе распределением часто называют не функцию
w⁢(x), а её интеграл W⁢(x)=∫w⁢(x)⁢𝑑x. Такую функцию в физике принято
называть интегральным или кумулятивным распределением. В англоязычной литературе
для этих функций принято использовать сокращения:
pdf (probability distribution function) и
cdf (cumulative distribution function)
соответственно.

Гистограммы.

Проиллюстрируем наглядно понятие плотности распределения. Результат
большого числа измерений случайной величины удобно представить с помощью
специального типа графика — гистограммы.
Для этого область значений x, размещённую на оси абсцисс, разобьём на
равные малые интервалы — «корзины» или «бины» (англ. bins)
некоторого размера h. По оси ординат будем откладывать долю измерений w,
результаты которых попадают в соответствующую корзину. А именно,
пусть k — номер корзины; nk — число измерений, попавших
в диапазон x∈[k⁢h,(k+1)⁢h]. Тогда на графике изобразим «столбик»
шириной h и высотой wk=nk/n.
В результате получим картину, подобную изображённой на рис. 2.1.

Рис. 2.1: Пример гистограммы для нормального распределения (x¯=10,
σ=1,0, h=0,1, n=104)

Высоты построенных столбиков будут приближённо соответствовать значению
плотности распределения w⁢(x) вблизи соответствующей точки x.
Если устремить число измерений к бесконечности (n→∞), а ширину корзин
к нулю (h→0), то огибающая гистограммы будет стремиться к некоторой
непрерывной функции w⁢(x).

Самые высокие столбики гистограммы будут группироваться вблизи максимума
функции w⁢(x) — это наиболее вероятное значение случайной величины.
Если отклонения в положительную и отрицательную стороны равновероятны,
то гистограмма будет симметрична — в таком случае среднее значение ⟨x⟩
также будет лежать вблизи этого максимума. Ширина гистограммы будет характеризовать разброс
значений случайной величины — по порядку величины
она, как правило, близка к среднеквадратичному отклонению sx.

Свойства распределений.

Из определения функции w⁢(x) следует, что вероятность получить в результате
эксперимента величину x в диапазоне от a до b
можно найти, вычислив интеграл:

Px∈[a,b]=∫abw⁢(x)⁢𝑑x. (2.1)

Согласно определению вероятности, сумма вероятностей для всех возможных случаев
всегда равна единице. Поэтому интеграл распределения w⁢(x) по всей области
значений x (то есть суммарная площадь под графиком w⁢(x)) равен единице:

Это соотношение называют условием нормировки.

Среднее и дисперсия.

Вычислим среднее по построенной гистограмме. Если размер корзин
h достаточно мал, все измерения в пределах одной корзины можно считать примерно
одинаковыми. Тогда среднее арифметическое всех результатов можно вычислить как

Переходя к пределу, получим следующее определение среднего значения
случайной величины:

где интегрирование ведётся по всей области значений x.
В теории вероятностей x¯ также называют математическим ожиданием
распределения.
Величину

σ2=(x-x¯)2¯=∫(x-x¯)2⁢w⁢𝑑x (2.3)

называют дисперсией распределения. Значение σ есть
срекднеквадратичное отклонение в пределе n→∞. Оно имеет ту
же размерность, что и сама величина x и характеризует разброс распределения.
Именно эту величину, как правило, приводят как характеристику погрешности
измерения x.

Доверительный интервал.

Обозначим как P|Δ⁢x|<δ вероятность
того, что отклонение от среднего Δ⁢x=x-x¯ составит величину,
не превосходящую по модулю значение δ:

P|Δ⁢x|<δ=∫x¯-δx¯+δw⁢(x)⁢𝑑x. (2.4)

Эту величину называют доверительной вероятностью для
доверительного интервала |x-x¯|≤δ.

2.2 Нормальное распределение

Одним из наиболее примечательных результатов теории вероятностей является
так называемая центральная предельная теорема. Она утверждает,
что сумма большого количества независимых случайных слагаемых, каждое
из которых вносит в эту сумму относительно малый вклад, подчиняется
универсальному закону, не зависимо от того, каким вероятностным законам
подчиняются её составляющие, — так называемому нормальному
распределению
(или распределению Гаусса).

Доказательство теоремы довольно громоздко и мы его не приводим (его можно найти
в любом учебнике по теории вероятностей). Остановимся
кратко на том, что такое нормальное распределение и его основных свойствах.

Плотность нормального распределения выражается следующей формулой:

w𝒩⁢(x)=12⁢π⁢σ⁢e-(x-x¯)22⁢σ2. (2.5)

Здесь x¯ и σ
— параметры нормального распределения: x¯ равно
среднему значению x, a σ —
среднеквадратичному отклонению, вычисленным в пределе n→∞.

Как видно из рис. 2.1, распределение представляет собой
симметричный
«колокол», положение вершины которого
соответствует x¯ (ввиду симметрии оно же
совпадает с наиболее вероятным значением — максимумом
функции w𝒩⁢(x)).

При значительном отклонении x от среднего величина
w𝒩⁢(x)
очень быстро убывает. Это означает, что вероятность встретить отклонения,
существенно большие, чем σ, оказывается пренебрежимо
мала
. Ширина «колокола» по порядку величины
равна σ — она характеризует «разброс»
экспериментальных данных относительно среднего значения.

Замечание. Точки x=x¯±σ являются точками
перегиба графика w⁢(x) (в них вторая производная по x
обращается в нуль, w′′=0), а их положение по высоте составляет
w⁢(x¯±σ)/w⁢(x¯)=e-1/2≈0,61
от высоты вершины.

Универсальный характер центральной предельной теоремы позволяет широко
применять на практике нормальное (гауссово) распределение для обработки
результатов измерений, поскольку часто случайные погрешности складываются из
множества случайных независимых факторов. Заметим, что на практике
для приближённой оценки параметров нормального распределения
случайной величины используются выборочные значения среднего
и дисперсии: x¯≈⟨x⟩, sx≈σx.

x-x0σ2=2w⁢(x)σ1=1

Рис. 2.2: Плотность нормального распределения

Доверительные вероятности.

Вычислим некоторые доверительные вероятности (2.4) для нормально
распределённых случайных величин.

Замечание. Значение интеграла вида ∫e-x2/2⁢𝑑x
(его называют интегралом ошибок) в элементарных функциях не выражается,
но легко находится численно.

Вероятность того, что результат отдельного измерения x окажется
в пределах x¯±σ оказывается равна

P|Δ⁢x|<σ=∫x¯-σx¯+σw𝒩⁢𝑑x≈0,68.

Вероятность отклонения в пределах x¯±2⁢σ:

а в пределах x¯±3⁢σ:

Иными словами, при большом числе измерений нормально распределённой
величины можно ожидать, что лишь треть измерений выпадут за пределы интервала
[x¯-σ,x¯+σ]. При этом около 5%
измерений выпадут за пределы [x¯-2⁢σ;x¯+2⁢σ],
и лишь 0,27% окажутся за пределами
[x¯-3⁢σ;x¯+3⁢σ].

Пример. В сообщениях об открытии бозона Хиггса на Большом адронном коллайдере
говорилось о том, что исследователи ждали подтверждение результатов
с точностью «5 сигма». Используя нормальное распределение (2.5)
нетрудно посчитать, что они использовали доверительную вероятность
P≈1-5,7⋅10-7=0,99999943. Такую точность можно назвать фантастической.

Полученные значения доверительных вероятностей используются при
стандартной записи результатов измерений. В физических измерениях
(в частности, в учебной лаборатории), как правило, используется P=0,68,
то есть, запись

означает, что измеренное значение лежит в диапазоне (доверительном
интервале) x∈[x¯-δ⁢x;x¯+δ⁢x] с
вероятностью 68%. Таким образом погрешность ±δ⁢x считается
равной одному среднеквадратичному отклонению: δ⁢x=σ.
В технических измерениях чаще используется P=0,95, то есть под
абсолютной погрешностью имеется в виду удвоенное среднеквадратичное
отклонение, δ⁢x=2⁢σ. Во избежание разночтений доверительную
вероятность следует указывать отдельно.

Замечание. Хотя нормальный закон распределения встречается на практике довольно
часто, стоит помнить, что он реализуется далеко не всегда.
Полученные выше соотношения для вероятностей попадания значений в
доверительные интервалы можно использовать в качестве простейшего
признака нормальности распределения: в частности, если количество попадающих
в интервал ±σ результатов существенно отличается от 2/3 — это повод
для более детального исследования закона распределения ошибок.

Сравнение результатов измерений.

Теперь мы можем дать количественный критерий для сравнения двух измеренных
величин или двух результатов измерения одной и той же величины.

Пусть x1 и x2 (x1≠x2) измерены с
погрешностями σ1 и σ2 соответственно.
Ясно, что если различие результатов |x2-x1| невелико,
его можно объяснить просто случайными отклонениями.
Если же теория предсказывает, что вероятность обнаружить такое отклонение
слишком мала, различие результатов следует признать значимым.
Предварительно необходимо договориться о соответствующем граничном значении
вероятности. Универсального значения здесь быть не может,
поэтому приходится полагаться на субъективный выбор исследователя. Часто
в качестве «разумной» границы выбирают вероятность 5%,
что, как видно из изложенного выше, для нормального распределения
соответствует отклонению более, чем на 2⁢σ.

Допустим, одна из величин известна с существенно большей точностью:
σ2≪σ1 (например, x1 — результат, полученный
студентом в лаборатории, x2 — справочное значение).
Поскольку σ2 мало, x2 можно принять за «истинное»:
x2≈x¯. Предполагая, что погрешность измерения
x1 подчиняется нормальному закону с и дисперсией σ12,
можно утверждать, что
различие считают будет значимы, если

Пусть погрешности измерений сравнимы по порядку величины:
σ1∼σ2. В теории вероятностей показывается, что
линейная комбинация нормально распределённых величин также имеет нормальное
распределение с дисперсией σ2=σ12+σ22
(см. также правила сложения погрешностей (2.7)). Тогда
для проверки гипотезы о том, что x1 и x2 являются измерениями
одной и той же величины, нужно вычислить, является ли значимым отклонение
|x1-x2| от нуля при σ=σ12+σ22.


Пример. Два студента получили следующие значения для теплоты испарения
некоторой жидкости: x1=40,3±0,2 кДж/моль и
x2=41,0±0,3 кДж/моль, где погрешность соответствует
одному стандартному отклонению. Можно ли утверждать, что они исследовали
одну и ту же жидкость?

Имеем наблюдаемую разность |x1-x2|=0,7 кДж/моль,
среднеквадратичное отклонение для разности
σ=0,22+0,32=0,36 кДж/моль.
Их отношение |x2-x1|σ≈2. Из
свойств нормального распределения находим вероятность того, что измерялась
одна и та же величина, а различия в ответах возникли из-за случайных
ошибок: P≈5%. Ответ на вопрос, «достаточно»
ли мала или велика эта вероятность, остаётся на усмотрение исследователя.

Замечание. Изложенные здесь соображения применимы, только если x¯ и
его стандартное отклонение σ получены на основании достаточно
большой выборки n≫1 (или заданы точно). При небольшом числе измерений
(n≲10) выборочные средние ⟨x⟩ и среднеквадратичное отклонение
sx сами имеют довольно большую ошибку, а
их распределение будет описываться не нормальным законом, а так
называемым t-распределением Стъюдента. В частности, в зависимости от
значения n интервал ⟨x⟩±sx будет соответствовать несколько
меньшей доверительной вероятности, чем P=0,68. Особенно резко различия
проявляются при высоких уровнях доверительных вероятностей P→1.

2.3 Независимые величины

Величины x и y называют независимыми если результат измерения одной
из них никак не влияет на результат измерения другой. Для таких величин вероятность того, что x окажется в некоторой области X, и одновременно y — в области Y,
равна произведению соответствующих вероятностей:

Обозначим отклонения величин от их средних как Δ⁢x=x-x¯ и
Δ⁢y=y-y¯.
Средние значения этих отклонений равны, очевидно, нулю: Δ⁢x¯=x¯-x¯=0,
Δ⁢y¯=0. Из независимости величин x и y следует,
что среднее значение от произведения Δ⁢x⋅Δ⁢y¯
равно произведению средних Δ⁢x¯⋅Δ⁢y¯
и, следовательно, равно нулю:

Δ⁢x⋅Δ⁢y¯=Δ⁢x¯⋅Δ⁢y¯=0. (2.6)

Пусть измеряемая величина z=x+y складывается из двух независимых
случайных слагаемых x и y, для которых известны средние
x¯ и y¯, и их среднеквадратичные погрешности
σx и σy. Непосредственно из определения (1.1)
следует, что среднее суммы равно сумме средних:

Найдём дисперсию σz2. В силу независимости имеем

Δ⁢z2¯=Δ⁢x2¯+Δ⁢y2¯+2⁢Δ⁢x⋅Δ⁢y¯≈Δ⁢x2¯+Δ⁢y2¯,

то есть:

Таким образом, при сложении независимых величин их погрешности
складываются среднеквадратичным образом.

Подчеркнём, что для справедливости соотношения (2.7)
величины x и y не обязаны быть нормально распределёнными —
достаточно существования конечных значений их дисперсий. Однако можно
показать, что если x и y распределены нормально, нормальным
будет и распределение их суммы
.

Замечание. Требование независимости
слагаемых является принципиальным. Например, положим y=x. Тогда
z=2⁢x. Здесь y и x, очевидно, зависят друг от друга. Используя
(2.7), находим σ2⁢x=2⁢σx,
что, конечно, неверно — непосредственно из определения
следует, что σ2⁢x=2⁢σx.

Отдельно стоит обсудить математическую структуру формулы (2.7).
Если одна из погрешностей много больше другой, например,
σx≫σy,
то меньшей погрешностью можно пренебречь, σx+y≈σx.
С другой стороны, если два источника погрешностей имеют один порядок
σx∼σy, то и σx+y∼σx∼σy.

Эти обстоятельства важны при планирования эксперимента: как правило,
величина, измеренная наименее точно, вносит наибольший вклад в погрешность
конечного результата. При этом, пока не устранены наиболее существенные
ошибки, бессмысленно гнаться за повышением точности измерения остальных
величин.

Пример. Пусть σy=σx/3,
тогда σz=σx⁢1+19≈1,05⁢σx,
то есть при различии двух погрешностей более, чем в 3 раза, поправка
к погрешности составляет менее 5%, и уже нет особого смысла в учёте
меньшей погрешности: σz≈σx. Это утверждение
касается сложения любых независимых источников погрешностей в эксперименте.

2.4 Погрешность среднего

Выборочное среднее арифметическое значение ⟨x⟩, найденное
по результатам n измерений, само является случайной величиной.
Действительно, если поставить серию одинаковых опытов по n измерений,
то в каждом опыте получится своё среднее значение, отличающееся от
предельного среднего x¯.

Вычислим среднеквадратичную погрешность среднего арифметического
σ⟨x⟩.
Рассмотрим вспомогательную сумму n слагаемых

Если {xi} есть набор независимых измерений
одной и той же физической величины, то мы можем, применяя результат
(2.7) предыдущего параграфа, записать

σZ=σx12+σx22+…+σxn2=n⁢σx,

поскольку под корнем находится n одинаковых слагаемых. Отсюда с
учётом ⟨x⟩=Z/n получаем

Таким образом, погрешность среднего значения x по результатам
n независимых измерений оказывается в n раз меньше погрешности
отдельного измерения
. Это один из важнейших результатов, позволяющий
уменьшать случайные погрешности эксперимента за счёт многократного
повторения измерений.

Подчеркнём отличия между σx и σ⟨x⟩:

величина σx — погрешность отдельного
измерения
— является характеристикой разброса значений
в совокупности измерений {xi}, i=1..n. При
нормальном законе распределения примерно 68% измерений попадают в
интервал ⟨x⟩±σx;

величина σ⟨x⟩ — погрешность
среднего
— характеризует точность, с которой определено
среднее значение измеряемой физической величины ⟨x⟩ относительно
предельного («истинного») среднего x¯;
при этом с доверительной вероятностью P=68% искомая величина x¯
лежит в интервале
⟨x⟩-σ⟨x⟩<x¯<⟨x⟩+σ⟨x⟩.

2.5 Результирующая погрешность опыта

Пусть для некоторого результата измерения известна оценка его максимальной
систематической погрешности Δсист и случайная
среднеквадратичная
погрешность σслуч. Какова «полная»
погрешность измерения?

Предположим для простоты, что измеряемая величина в принципе
может быть определена сколь угодно точно, так что можно говорить о
некотором её «истинном» значении xист
(иными словами, погрешность результата связана в основном именно с
процессом измерения). Назовём полной погрешностью измерения
среднеквадратичное значения отклонения от результата измерения от
«истинного»:

Отклонение x-xист можно представить как сумму случайного
отклонения от среднего δ⁢xслуч=x-x¯
и постоянной (но, вообще говоря, неизвестной) систематической составляющей
δ⁢xсист=x¯-xист=const:

Причём случайную составляющую можно считать независимой от систематической.
В таком случае из (2.7) находим:

σполн2=⟨δ⁢xсист2⟩+⟨δ⁢xслуч2⟩≤Δсист2+σслуч2. (2.9)

Таким образом, для получения максимального значения полной
погрешности некоторого измерения нужно квадратично сложить максимальную
систематическую и случайную погрешности.

Если измерения проводятся многократно, то согласно (2.8)
случайная составляющая погрешности может быть уменьшена, а систематическая
составляющая при этом остаётся неизменной:

Отсюда следует важное практическое правило
(см. также обсуждение в п. 2.3): если случайная погрешность измерений
в 2–3 раза меньше предполагаемой систематической, то
нет смысла проводить многократные измерения в попытке уменьшить погрешность
всего эксперимента. В такой ситуации измерения достаточно повторить
2–3 раза — чтобы убедиться в повторяемости результата, исключить промахи
и проверить, что случайная ошибка действительно мала.
В противном случае повторение измерений может иметь смысл до
тех пор, пока погрешность среднего
σ⟨x⟩=σxn
не станет меньше систематической.


Замечание. Поскольку конкретная
величина систематической погрешности, как правило, не известна, её
можно в некотором смысле рассматривать наравне со случайной —
предположить, что её величина была определена по некоторому случайному
закону перед началом измерений (например, при изготовлении линейки
на заводе произошло некоторое случайное искажение шкалы). При такой
трактовке формулу (2.9) можно рассматривать просто
как частный случай формулы сложения погрешностей независимых величин
(2.7).

Подчеркнем, что вероятностный закон, которому подчиняется
систематическая ошибка, зачастую неизвестен. Поэтому неизвестно и
распределение итогового результата. Из этого, в частности, следует,
что мы не можем приписать интервалу x±Δсист какую-либо
определённую доверительную вероятность — она равна 0,68
только если систематическая ошибка имеет нормальное распределение.
Можно, конечно, предположить,
— и так часто делают — что, к примеру, ошибки
при изготовлении линеек на заводе имеют гауссов характер. Также часто
предполагают, что систематическая ошибка имеет равномерное
распределение (то есть «истинное» значение может с равной вероятностью
принять любое значение в пределах интервала ±Δсист).
Строго говоря, для этих предположений нет достаточных оснований.


Пример. В результате измерения диаметра проволоки микрометрическим винтом,
имеющим цену деления h=0,01 мм, получен следующий набор из n=8 значений:

Вычисляем среднее значение: ⟨d⟩≈386,3 мкм.
Среднеквадратичное отклонение:
σd≈9,2 мкм. Случайная погрешность среднего согласно
(2.8):
σ⟨d⟩=σd8≈3,2
мкм. Все результаты лежат в пределах ±2⁢σd, поэтому нет
причин сомневаться в нормальности распределения. Максимальную погрешность
микрометра оценим как половину цены деления, Δ=h2=5 мкм.
Результирующая полная погрешность
σ≤Δ2+σd28≈6,0 мкм.
Видно, что σслуч≈Δсист и проводить дополнительные измерения
особого смысла нет. Окончательно результат измерений может быть представлен
в виде (см. также правила округления
результатов измерений в п. 4.3.2)



d=386±6⁢мкм,εd=1,5%.

Заметим, что поскольку случайная погрешность и погрешность
прибора здесь имеют один порядок величины, наблюдаемый случайный разброс
данных может быть связан как с неоднородностью сечения проволоки,
так и с дефектами микрометра (например, с неровностями зажимов, люфтом
винта, сухим трением, деформацией проволоки под действием микрометра
и т. п.). Для ответа на вопрос, что именно вызвало разброс, требуются
дополнительные исследования, желательно с использованием более точных
приборов.


Пример. Измерение скорости
полёта пули было осуществлено с погрешностью δ⁢v=±1 м/c.
Результаты измерений для n=6 выстрелов представлены в таблице:

Усреднённый результат ⟨v⟩=162,0⁢м/с,
среднеквадратичное отклонение σv=13,8⁢м/c, случайная
ошибка для средней скорости
σv¯=σv/6=5,6⁢м/с.
Поскольку разброс экспериментальных данных существенно превышает погрешность
каждого измерения, σv≫δ⁢v, он почти наверняка связан
с реальным различием скоростей пули в разных выстрелах, а не с ошибками
измерений. В качестве результата эксперимента представляют интерес
как среднее значение скоростей ⟨v⟩=162±6⁢м/с
(ε≈4%), так и значение σv≈14⁢м/с,
характеризующее разброс значений скоростей от выстрела к выстрелу.
Малая инструментальная погрешность в принципе позволяет более точно
измерить среднее и дисперсию, и исследовать закон распределения выстрелов
по скоростям более детально — для этого требуется набрать
бо́льшую статистику по выстрелам.


Пример. Измерение скорости
полёта пули было осуществлено с погрешностью δ⁢v=10 м/c. Результаты
измерений для n=6 выстрелов представлены в таблице:

Усреднённый результат ⟨v⟩=163,3⁢м/с,
σv=12,1⁢м/c, σ⟨v⟩=5⁢м/с,
σполн≈11,2⁢м/с. Инструментальная
погрешность каждого измерения превышает разброс данных, поэтому в
этом опыте затруднительно сделать вывод о различии скоростей от выстрела
к выстрелу. Результат измерений скорости пули:
⟨v⟩=163±11⁢м/с,
ε≈7%. Проводить дополнительные выстрелы при такой
большой инструментальной погрешности особого смысла нет —
лучше поработать над точностью приборов и методикой измерений.

2.6 Обработка косвенных измерений

Косвенными называют измерения, полученные в результате расчётов,
использующих результаты прямых (то есть «непосредственных»)
измерений физических величин. Сформулируем основные правила пересчёта
погрешностей при косвенных измерениях.

2.6.1 Случай одной переменной

Пусть в эксперименте измеряется величина x, а её «наилучшее»
(в некотором смысле) значение равно x⋆ и оно известно с
погрешностью σx. После чего с помощью известной функции
вычисляется величина y=f⁢(x).

В качестве «наилучшего» приближения для y используем значение функции
при «наилучшем» x:

Найдём величину погрешности σy. Обозначая отклонение измеряемой
величины как Δ⁢x=x-x⋆, и пользуясь определением производной,
при условии, что функция y⁢(x) — гладкая
вблизи x≈x⋆, запишем

где f′≡d⁢yd⁢x — производная фукнции f⁢(x), взятая в точке
x⋆. Возведём полученное в квадрат, проведём усреднение
(σy2=⟨Δ⁢y2⟩,
σx2=⟨Δ⁢x2⟩), и затем снова извлечём
корень. В результате получим


Пример. Для степенной функции
y=A⁢xn имеем σy=n⁢A⁢xn-1⁢σx, откуда



σyy=n⁢σxx,или  εy=n⁢εx,

то есть относительная погрешность степенной функции возрастает пропорционально
показателю степени n.

Пример. Для y=1/x имеем ε1/x=εx
— при обращении величины сохраняется её относительная
погрешность.

Упражнение. Найдите погрешность логарифма y=ln⁡x, если известны x
и σx.

Упражнение. Найдите погрешность показательной функции y=ax,
если известны x и σx. Коэффициент a задан точно.

2.6.2 Случай многих переменных

Пусть величина u вычисляется по измеренным значениям нескольких
различных независимых физических величин x, y, …
на основе известного закона u=f⁢(x,y,…). В качестве
наилучшего значения можно по-прежнему взять значение функции f
при наилучших значениях измеряемых параметров:

Для нахождения погрешности σu воспользуемся свойством,
известным из математического анализа, — малые приращения гладких
функции многих переменных складываются линейно, то есть справедлив
принцип суперпозиции малых приращений:

где символом fx′≡∂⁡f∂⁡x обозначена
частная производная функции f по переменной x —
то есть обычная производная f по x, взятая при условии, что
все остальные аргументы (кроме x) считаются постоянными параметрами.
Тогда пользуясь формулой для нахождения дисперсии суммы независимых
величин (2.7), получим соотношение, позволяющее вычислять
погрешности косвенных измерений для произвольной функции
u=f⁢(x,y,…):

σu2=fx′⁣2⁢σx2+fy′⁣2⁢σy2+… (2.11)

Это и есть искомая общая формула пересчёта погрешностей при косвенных
измерениях.

Отметим, что формулы (2.10) и (2.11) применимы
только если относительные отклонения всех величин малы
(εx,εy,…≪1),
а измерения проводятся вдали от особых точек функции f (производные
fx′, fy′ … не должны обращаться в бесконечность).
Также подчеркнём, что все полученные здесь формулы справедливы только
для независимых переменных x, y, …

Остановимся на некоторых важных частных случаях формулы
(2.11).


Пример. Для суммы (или разности) u=∑i=1nai⁢xi имеем



σu2=∑i=1nai2⁢σxi2.

(2.12)


Пример. Найдём погрешность степенной функции:
u=xα⋅yβ⋅…. Тогда нетрудно получить,
что



σu2u2=α2⁢σx2x2+β2⁢σy2y2+…

или через относительные погрешности



εu2=α2⁢εx2+β2⁢εy2+…

(2.13)


Пример. Вычислим погрешность произведения и частного: u=x⁢y или u=x/y.
Тогда в обоих случаях имеем



εu2=εx2+εy2,

(2.14)

то есть при умножении или делении относительные погрешности складываются
квадратично.


Пример. Рассмотрим несколько более сложный случай: нахождение угла по его тангенсу



u=arctgyx.

В таком случае, пользуясь тем, что (arctgz)′=11+z2,
где z=y/x, и используя производную сложной функции, находим
ux′=uz′⁢zx′=-yx2+y2,
uy′=uz′⁢zy′=xx2+y2, и наконец



σu2=y2⁢σx2+x2⁢σy2(x2+y2)2.

Упражнение. Найти погрешность вычисления гипотенузы z=x2+y2
прямоугольного треугольника по измеренным катетам x и y.

По итогам данного раздела можно дать следующие практические рекомендации.

  • Как правило, нет смысла увеличивать точность измерения какой-то одной
    величины, если другие величины, используемые в расчётах, остаются
    измеренными относительно грубо — всё равно итоговая погрешность
    скорее всего будет определяться самым неточным измерением. Поэтому
    все измерения имеет смысл проводить примерно с одной и той же
    относительной погрешностью
    .

  • При этом, как следует из (2.13), особое внимание
    следует уделять измерению величин, возводимых при расчётах в степени
    с большими показателями. А при сложных функциональных зависимостях
    имеет смысл детально проанализировать структуру формулы
    (2.11):
    если вклад от некоторой величины в общую погрешность мал, нет смысла
    гнаться за высокой точностью её измерения, и наоборот, точность некоторых
    измерений может оказаться критически важной.

  • Следует избегать измерения малых величин как разности двух близких
    значений (например, толщины стенки цилиндра как разности внутреннего
    и внешнего радиусов): если u=x-y, то абсолютная погрешность
    σu=σx2+σy2
    меняется мало, однако относительная погрешность
    εu=σux-y
    может оказаться неприемлемо большой, если x≈y.

Результат любого измерения не определён однозначно и имеет случайную составляющую.
Поэтому адекватным языком для описания погрешностей является язык вероятностей.
Тот факт, что значение некоторой величины «случайно», не означает, что
она может принимать совершенно произвольные значения. Ясно, что частоты, с которыми
возникает те или иные значения, различны. Вероятностные законы, которым
подчиняются случайные величины, называют распределениями.

2.1 Случайная величина

Случайной будем называть величину, значение которой не может быть достоверно определено экспериментатором. Чаще всего подразумевается, что случайная величина будет изменяться при многократном повторении одного и того же эксперимента. При интерпретации результатов измерений в физических экспериментах, обычно случайными также считаются величины, значение которых является фиксированным, но не известно экспериментатору. Например смещение нуля шкалы прибора. Для формализации работы со случайными величинами используют понятие вероятности. Численное значение вероятности того, что какая-то величина примет то или иное значение определяется либо как относительная частота наблюдения того или иного значения при повторении опыта большое количество раз, либо как оценка на основе данных других экспериментов.

Замечание. 
Хотя понятия вероятности и случайной величины являются основополагающими, в литературе нет единства в их определении. Обсуждение формальных тонкостей или построение строгой теории лежит за пределами данного пособия. Поэтому на начальном этапе лучше использовать «интуитивное» понимание этих сущностей. Заинтересованным читателям рекомендуем обратиться к специальной литературе: [5].

Рассмотрим случайную физическую величину x, которая при измерениях может
принимать непрерывный набор значений. Пусть
P[x0,x0+δ⁢x] — вероятность того, что результат окажется вблизи
некоторой точки x0 в пределах интервала δ⁢x: x∈[x0,x0+δ⁢x].
Устремим интервал
δ⁢x к нулю. Нетрудно понять, что вероятность попасть в этот интервал
также будет стремиться к нулю. Однако отношение
w⁢(x0)=P[x0,x0+δ⁢x]δ⁢x будет оставаться конечным.
Функцию w⁢(x) называют плотностью распределения вероятности или кратко
распределением непрерывной случайной величины x.

Замечание. В математической литературе распределением часто называют не функцию
w⁢(x), а её интеграл W⁢(x)=∫w⁢(x)⁢𝑑x. Такую функцию в физике принято
называть интегральным или кумулятивным распределением. В англоязычной литературе
для этих функций принято использовать сокращения:
pdf (probability distribution function) и
cdf (cumulative distribution function)
соответственно.

Гистограммы.

Проиллюстрируем наглядно понятие плотности распределения. Результат
большого числа измерений случайной величины удобно представить с помощью
специального типа графика — гистограммы.
Для этого область значений x, размещённую на оси абсцисс, разобьём на
равные малые интервалы — «корзины» или «бины» (англ. bins)
некоторого размера h. По оси ординат будем откладывать долю измерений w,
результаты которых попадают в соответствующую корзину. А именно,
пусть k — номер корзины; nk — число измерений, попавших
в диапазон x∈[k⁢h,(k+1)⁢h]. Тогда на графике изобразим «столбик»
шириной h и высотой wk=nk/n.
В результате получим картину, подобную изображённой на рис. 2.1.

Рис. 2.1: Пример гистограммы для нормального распределения (x¯=10,
σ=1,0, h=0,1, n=104)

Высоты построенных столбиков будут приближённо соответствовать значению
плотности распределения w⁢(x) вблизи соответствующей точки x.
Если устремить число измерений к бесконечности (n→∞), а ширину корзин
к нулю (h→0), то огибающая гистограммы будет стремиться к некоторой
непрерывной функции w⁢(x).

Самые высокие столбики гистограммы будут группироваться вблизи максимума
функции w⁢(x) — это наиболее вероятное значение случайной величины.
Если отклонения в положительную и отрицательную стороны равновероятны,
то гистограмма будет симметрична — в таком случае среднее значение ⟨x⟩
также будет лежать вблизи этого максимума. Ширина гистограммы будет характеризовать разброс
значений случайной величины — по порядку величины
она, как правило, близка к среднеквадратичному отклонению sx.

Свойства распределений.

Из определения функции w⁢(x) следует, что вероятность получить в результате
эксперимента величину x в диапазоне от a до b
можно найти, вычислив интеграл:

Px∈[a,b]=∫abw⁢(x)⁢𝑑x. (2.1)

Согласно определению вероятности, сумма вероятностей для всех возможных случаев
всегда равна единице. Поэтому интеграл распределения w⁢(x) по всей области
значений x (то есть суммарная площадь под графиком w⁢(x)) равен единице:

Это соотношение называют условием нормировки.

Среднее и дисперсия.

Вычислим среднее по построенной гистограмме. Если размер корзин
h достаточно мал, все измерения в пределах одной корзины можно считать примерно
одинаковыми. Тогда среднее арифметическое всех результатов можно вычислить как

Переходя к пределу, получим следующее определение среднего значения
случайной величины:

где интегрирование ведётся по всей области значений x.
В теории вероятностей x¯ также называют математическим ожиданием
распределения.
Величину

σ2=(x-x¯)2¯=∫(x-x¯)2⁢w⁢𝑑x (2.3)

называют дисперсией распределения. Значение σ есть
срекднеквадратичное отклонение в пределе n→∞. Оно имеет ту
же размерность, что и сама величина x и характеризует разброс распределения.
Именно эту величину, как правило, приводят как характеристику погрешности
измерения x.

Доверительный интервал.

Обозначим как P|Δ⁢x|<δ вероятность
того, что отклонение от среднего Δ⁢x=x-x¯ составит величину,
не превосходящую по модулю значение δ:

P|Δ⁢x|<δ=∫x¯-δx¯+δw⁢(x)⁢𝑑x. (2.4)

Эту величину называют доверительной вероятностью для
доверительного интервала |x-x¯|≤δ.

2.2 Нормальное распределение

Одним из наиболее примечательных результатов теории вероятностей является
так называемая центральная предельная теорема. Она утверждает,
что сумма большого количества независимых случайных слагаемых, каждое
из которых вносит в эту сумму относительно малый вклад, подчиняется
универсальному закону, не зависимо от того, каким вероятностным законам
подчиняются её составляющие, — так называемому нормальному
распределению
(или распределению Гаусса).

Доказательство теоремы довольно громоздко и мы его не приводим (его можно найти
в любом учебнике по теории вероятностей). Остановимся
кратко на том, что такое нормальное распределение и его основных свойствах.

Плотность нормального распределения выражается следующей формулой:

w𝒩⁢(x)=12⁢π⁢σ⁢e-(x-x¯)22⁢σ2. (2.5)

Здесь x¯ и σ
— параметры нормального распределения: x¯ равно
среднему значению x, a σ —
среднеквадратичному отклонению, вычисленным в пределе n→∞.

Как видно из рис. 2.1, распределение представляет собой
симметричный
«колокол», положение вершины которого
соответствует x¯ (ввиду симметрии оно же
совпадает с наиболее вероятным значением — максимумом
функции w𝒩⁢(x)).

При значительном отклонении x от среднего величина
w𝒩⁢(x)
очень быстро убывает. Это означает, что вероятность встретить отклонения,
существенно большие, чем σ, оказывается пренебрежимо
мала
. Ширина «колокола» по порядку величины
равна σ — она характеризует «разброс»
экспериментальных данных относительно среднего значения.

Замечание. Точки x=x¯±σ являются точками
перегиба графика w⁢(x) (в них вторая производная по x
обращается в нуль, w′′=0), а их положение по высоте составляет
w⁢(x¯±σ)/w⁢(x¯)=e-1/2≈0,61
от высоты вершины.

Универсальный характер центральной предельной теоремы позволяет широко
применять на практике нормальное (гауссово) распределение для обработки
результатов измерений, поскольку часто случайные погрешности складываются из
множества случайных независимых факторов. Заметим, что на практике
для приближённой оценки параметров нормального распределения
случайной величины используются выборочные значения среднего
и дисперсии: x¯≈⟨x⟩, sx≈σx.

x-x0σ2=2w⁢(x)σ1=1

Рис. 2.2: Плотность нормального распределения

Доверительные вероятности.

Вычислим некоторые доверительные вероятности (2.4) для нормально
распределённых случайных величин.

Замечание. Значение интеграла вида ∫e-x2/2⁢𝑑x
(его называют интегралом ошибок) в элементарных функциях не выражается,
но легко находится численно.

Вероятность того, что результат отдельного измерения x окажется
в пределах x¯±σ оказывается равна

P|Δ⁢x|<σ=∫x¯-σx¯+σw𝒩⁢𝑑x≈0,68.

Вероятность отклонения в пределах x¯±2⁢σ:

а в пределах x¯±3⁢σ:

Иными словами, при большом числе измерений нормально распределённой
величины можно ожидать, что лишь треть измерений выпадут за пределы интервала
[x¯-σ,x¯+σ]. При этом около 5%
измерений выпадут за пределы [x¯-2⁢σ;x¯+2⁢σ],
и лишь 0,27% окажутся за пределами
[x¯-3⁢σ;x¯+3⁢σ].

Пример. В сообщениях об открытии бозона Хиггса на Большом адронном коллайдере
говорилось о том, что исследователи ждали подтверждение результатов
с точностью «5 сигма». Используя нормальное распределение (2.5)
нетрудно посчитать, что они использовали доверительную вероятность
P≈1-5,7⋅10-7=0,99999943. Такую точность можно назвать фантастической.

Полученные значения доверительных вероятностей используются при
стандартной записи результатов измерений. В физических измерениях
(в частности, в учебной лаборатории), как правило, используется P=0,68,
то есть, запись

означает, что измеренное значение лежит в диапазоне (доверительном
интервале) x∈[x¯-δ⁢x;x¯+δ⁢x] с
вероятностью 68%. Таким образом погрешность ±δ⁢x считается
равной одному среднеквадратичному отклонению: δ⁢x=σ.
В технических измерениях чаще используется P=0,95, то есть под
абсолютной погрешностью имеется в виду удвоенное среднеквадратичное
отклонение, δ⁢x=2⁢σ. Во избежание разночтений доверительную
вероятность следует указывать отдельно.

Замечание. Хотя нормальный закон распределения встречается на практике довольно
часто, стоит помнить, что он реализуется далеко не всегда.
Полученные выше соотношения для вероятностей попадания значений в
доверительные интервалы можно использовать в качестве простейшего
признака нормальности распределения: в частности, если количество попадающих
в интервал ±σ результатов существенно отличается от 2/3 — это повод
для более детального исследования закона распределения ошибок.

Сравнение результатов измерений.

Теперь мы можем дать количественный критерий для сравнения двух измеренных
величин или двух результатов измерения одной и той же величины.

Пусть x1 и x2 (x1≠x2) измерены с
погрешностями σ1 и σ2 соответственно.
Ясно, что если различие результатов |x2-x1| невелико,
его можно объяснить просто случайными отклонениями.
Если же теория предсказывает, что вероятность обнаружить такое отклонение
слишком мала, различие результатов следует признать значимым.
Предварительно необходимо договориться о соответствующем граничном значении
вероятности. Универсального значения здесь быть не может,
поэтому приходится полагаться на субъективный выбор исследователя. Часто
в качестве «разумной» границы выбирают вероятность 5%,
что, как видно из изложенного выше, для нормального распределения
соответствует отклонению более, чем на 2⁢σ.

Допустим, одна из величин известна с существенно большей точностью:
σ2≪σ1 (например, x1 — результат, полученный
студентом в лаборатории, x2 — справочное значение).
Поскольку σ2 мало, x2 можно принять за «истинное»:
x2≈x¯. Предполагая, что погрешность измерения
x1 подчиняется нормальному закону с и дисперсией σ12,
можно утверждать, что
различие считают будет значимы, если

Пусть погрешности измерений сравнимы по порядку величины:
σ1∼σ2. В теории вероятностей показывается, что
линейная комбинация нормально распределённых величин также имеет нормальное
распределение с дисперсией σ2=σ12+σ22
(см. также правила сложения погрешностей (2.7)). Тогда
для проверки гипотезы о том, что x1 и x2 являются измерениями
одной и той же величины, нужно вычислить, является ли значимым отклонение
|x1-x2| от нуля при σ=σ12+σ22.


Пример. Два студента получили следующие значения для теплоты испарения
некоторой жидкости: x1=40,3±0,2 кДж/моль и
x2=41,0±0,3 кДж/моль, где погрешность соответствует
одному стандартному отклонению. Можно ли утверждать, что они исследовали
одну и ту же жидкость?

Имеем наблюдаемую разность |x1-x2|=0,7 кДж/моль,
среднеквадратичное отклонение для разности
σ=0,22+0,32=0,36 кДж/моль.
Их отношение |x2-x1|σ≈2. Из
свойств нормального распределения находим вероятность того, что измерялась
одна и та же величина, а различия в ответах возникли из-за случайных
ошибок: P≈5%. Ответ на вопрос, «достаточно»
ли мала или велика эта вероятность, остаётся на усмотрение исследователя.

Замечание. Изложенные здесь соображения применимы, только если x¯ и
его стандартное отклонение σ получены на основании достаточно
большой выборки n≫1 (или заданы точно). При небольшом числе измерений
(n≲10) выборочные средние ⟨x⟩ и среднеквадратичное отклонение
sx сами имеют довольно большую ошибку, а
их распределение будет описываться не нормальным законом, а так
называемым t-распределением Стъюдента. В частности, в зависимости от
значения n интервал ⟨x⟩±sx будет соответствовать несколько
меньшей доверительной вероятности, чем P=0,68. Особенно резко различия
проявляются при высоких уровнях доверительных вероятностей P→1.

2.3 Независимые величины

Величины x и y называют независимыми если результат измерения одной
из них никак не влияет на результат измерения другой. Для таких величин вероятность того, что x окажется в некоторой области X, и одновременно y — в области Y,
равна произведению соответствующих вероятностей:

Обозначим отклонения величин от их средних как Δ⁢x=x-x¯ и
Δ⁢y=y-y¯.
Средние значения этих отклонений равны, очевидно, нулю: Δ⁢x¯=x¯-x¯=0,
Δ⁢y¯=0. Из независимости величин x и y следует,
что среднее значение от произведения Δ⁢x⋅Δ⁢y¯
равно произведению средних Δ⁢x¯⋅Δ⁢y¯
и, следовательно, равно нулю:

Δ⁢x⋅Δ⁢y¯=Δ⁢x¯⋅Δ⁢y¯=0. (2.6)

Пусть измеряемая величина z=x+y складывается из двух независимых
случайных слагаемых x и y, для которых известны средние
x¯ и y¯, и их среднеквадратичные погрешности
σx и σy. Непосредственно из определения (1.1)
следует, что среднее суммы равно сумме средних:

Найдём дисперсию σz2. В силу независимости имеем

Δ⁢z2¯=Δ⁢x2¯+Δ⁢y2¯+2⁢Δ⁢x⋅Δ⁢y¯≈Δ⁢x2¯+Δ⁢y2¯,

то есть:

Таким образом, при сложении независимых величин их погрешности
складываются среднеквадратичным образом.

Подчеркнём, что для справедливости соотношения (2.7)
величины x и y не обязаны быть нормально распределёнными —
достаточно существования конечных значений их дисперсий. Однако можно
показать, что если x и y распределены нормально, нормальным
будет и распределение их суммы
.

Замечание. Требование независимости
слагаемых является принципиальным. Например, положим y=x. Тогда
z=2⁢x. Здесь y и x, очевидно, зависят друг от друга. Используя
(2.7), находим σ2⁢x=2⁢σx,
что, конечно, неверно — непосредственно из определения
следует, что σ2⁢x=2⁢σx.

Отдельно стоит обсудить математическую структуру формулы (2.7).
Если одна из погрешностей много больше другой, например,
σx≫σy,
то меньшей погрешностью можно пренебречь, σx+y≈σx.
С другой стороны, если два источника погрешностей имеют один порядок
σx∼σy, то и σx+y∼σx∼σy.

Эти обстоятельства важны при планирования эксперимента: как правило,
величина, измеренная наименее точно, вносит наибольший вклад в погрешность
конечного результата. При этом, пока не устранены наиболее существенные
ошибки, бессмысленно гнаться за повышением точности измерения остальных
величин.

Пример. Пусть σy=σx/3,
тогда σz=σx⁢1+19≈1,05⁢σx,
то есть при различии двух погрешностей более, чем в 3 раза, поправка
к погрешности составляет менее 5%, и уже нет особого смысла в учёте
меньшей погрешности: σz≈σx. Это утверждение
касается сложения любых независимых источников погрешностей в эксперименте.

2.4 Погрешность среднего

Выборочное среднее арифметическое значение ⟨x⟩, найденное
по результатам n измерений, само является случайной величиной.
Действительно, если поставить серию одинаковых опытов по n измерений,
то в каждом опыте получится своё среднее значение, отличающееся от
предельного среднего x¯.

Вычислим среднеквадратичную погрешность среднего арифметического
σ⟨x⟩.
Рассмотрим вспомогательную сумму n слагаемых

Если {xi} есть набор независимых измерений
одной и той же физической величины, то мы можем, применяя результат
(2.7) предыдущего параграфа, записать

σZ=σx12+σx22+…+σxn2=n⁢σx,

поскольку под корнем находится n одинаковых слагаемых. Отсюда с
учётом ⟨x⟩=Z/n получаем

Таким образом, погрешность среднего значения x по результатам
n независимых измерений оказывается в n раз меньше погрешности
отдельного измерения
. Это один из важнейших результатов, позволяющий
уменьшать случайные погрешности эксперимента за счёт многократного
повторения измерений.

Подчеркнём отличия между σx и σ⟨x⟩:

величина σx — погрешность отдельного
измерения
— является характеристикой разброса значений
в совокупности измерений {xi}, i=1..n. При
нормальном законе распределения примерно 68% измерений попадают в
интервал ⟨x⟩±σx;

величина σ⟨x⟩ — погрешность
среднего
— характеризует точность, с которой определено
среднее значение измеряемой физической величины ⟨x⟩ относительно
предельного («истинного») среднего x¯;
при этом с доверительной вероятностью P=68% искомая величина x¯
лежит в интервале
⟨x⟩-σ⟨x⟩<x¯<⟨x⟩+σ⟨x⟩.

2.5 Результирующая погрешность опыта

Пусть для некоторого результата измерения известна оценка его максимальной
систематической погрешности Δсист и случайная
среднеквадратичная
погрешность σслуч. Какова «полная»
погрешность измерения?

Предположим для простоты, что измеряемая величина в принципе
может быть определена сколь угодно точно, так что можно говорить о
некотором её «истинном» значении xист
(иными словами, погрешность результата связана в основном именно с
процессом измерения). Назовём полной погрешностью измерения
среднеквадратичное значения отклонения от результата измерения от
«истинного»:

Отклонение x-xист можно представить как сумму случайного
отклонения от среднего δ⁢xслуч=x-x¯
и постоянной (но, вообще говоря, неизвестной) систематической составляющей
δ⁢xсист=x¯-xист=const:

Причём случайную составляющую можно считать независимой от систематической.
В таком случае из (2.7) находим:

σполн2=⟨δ⁢xсист2⟩+⟨δ⁢xслуч2⟩≤Δсист2+σслуч2. (2.9)

Таким образом, для получения максимального значения полной
погрешности некоторого измерения нужно квадратично сложить максимальную
систематическую и случайную погрешности.

Если измерения проводятся многократно, то согласно (2.8)
случайная составляющая погрешности может быть уменьшена, а систематическая
составляющая при этом остаётся неизменной:

Отсюда следует важное практическое правило
(см. также обсуждение в п. 2.3): если случайная погрешность измерений
в 2–3 раза меньше предполагаемой систематической, то
нет смысла проводить многократные измерения в попытке уменьшить погрешность
всего эксперимента. В такой ситуации измерения достаточно повторить
2–3 раза — чтобы убедиться в повторяемости результата, исключить промахи
и проверить, что случайная ошибка действительно мала.
В противном случае повторение измерений может иметь смысл до
тех пор, пока погрешность среднего
σ⟨x⟩=σxn
не станет меньше систематической.


Замечание. Поскольку конкретная
величина систематической погрешности, как правило, не известна, её
можно в некотором смысле рассматривать наравне со случайной —
предположить, что её величина была определена по некоторому случайному
закону перед началом измерений (например, при изготовлении линейки
на заводе произошло некоторое случайное искажение шкалы). При такой
трактовке формулу (2.9) можно рассматривать просто
как частный случай формулы сложения погрешностей независимых величин
(2.7).

Подчеркнем, что вероятностный закон, которому подчиняется
систематическая ошибка, зачастую неизвестен. Поэтому неизвестно и
распределение итогового результата. Из этого, в частности, следует,
что мы не можем приписать интервалу x±Δсист какую-либо
определённую доверительную вероятность — она равна 0,68
только если систематическая ошибка имеет нормальное распределение.
Можно, конечно, предположить,
— и так часто делают — что, к примеру, ошибки
при изготовлении линеек на заводе имеют гауссов характер. Также часто
предполагают, что систематическая ошибка имеет равномерное
распределение (то есть «истинное» значение может с равной вероятностью
принять любое значение в пределах интервала ±Δсист).
Строго говоря, для этих предположений нет достаточных оснований.


Пример. В результате измерения диаметра проволоки микрометрическим винтом,
имеющим цену деления h=0,01 мм, получен следующий набор из n=8 значений:

Вычисляем среднее значение: ⟨d⟩≈386,3 мкм.
Среднеквадратичное отклонение:
σd≈9,2 мкм. Случайная погрешность среднего согласно
(2.8):
σ⟨d⟩=σd8≈3,2
мкм. Все результаты лежат в пределах ±2⁢σd, поэтому нет
причин сомневаться в нормальности распределения. Максимальную погрешность
микрометра оценим как половину цены деления, Δ=h2=5 мкм.
Результирующая полная погрешность
σ≤Δ2+σd28≈6,0 мкм.
Видно, что σслуч≈Δсист и проводить дополнительные измерения
особого смысла нет. Окончательно результат измерений может быть представлен
в виде (см. также правила округления
результатов измерений в п. 4.3.2)



d=386±6⁢мкм,εd=1,5%.

Заметим, что поскольку случайная погрешность и погрешность
прибора здесь имеют один порядок величины, наблюдаемый случайный разброс
данных может быть связан как с неоднородностью сечения проволоки,
так и с дефектами микрометра (например, с неровностями зажимов, люфтом
винта, сухим трением, деформацией проволоки под действием микрометра
и т. п.). Для ответа на вопрос, что именно вызвало разброс, требуются
дополнительные исследования, желательно с использованием более точных
приборов.


Пример. Измерение скорости
полёта пули было осуществлено с погрешностью δ⁢v=±1 м/c.
Результаты измерений для n=6 выстрелов представлены в таблице:

Усреднённый результат ⟨v⟩=162,0⁢м/с,
среднеквадратичное отклонение σv=13,8⁢м/c, случайная
ошибка для средней скорости
σv¯=σv/6=5,6⁢м/с.
Поскольку разброс экспериментальных данных существенно превышает погрешность
каждого измерения, σv≫δ⁢v, он почти наверняка связан
с реальным различием скоростей пули в разных выстрелах, а не с ошибками
измерений. В качестве результата эксперимента представляют интерес
как среднее значение скоростей ⟨v⟩=162±6⁢м/с
(ε≈4%), так и значение σv≈14⁢м/с,
характеризующее разброс значений скоростей от выстрела к выстрелу.
Малая инструментальная погрешность в принципе позволяет более точно
измерить среднее и дисперсию, и исследовать закон распределения выстрелов
по скоростям более детально — для этого требуется набрать
бо́льшую статистику по выстрелам.


Пример. Измерение скорости
полёта пули было осуществлено с погрешностью δ⁢v=10 м/c. Результаты
измерений для n=6 выстрелов представлены в таблице:

Усреднённый результат ⟨v⟩=163,3⁢м/с,
σv=12,1⁢м/c, σ⟨v⟩=5⁢м/с,
σполн≈11,2⁢м/с. Инструментальная
погрешность каждого измерения превышает разброс данных, поэтому в
этом опыте затруднительно сделать вывод о различии скоростей от выстрела
к выстрелу. Результат измерений скорости пули:
⟨v⟩=163±11⁢м/с,
ε≈7%. Проводить дополнительные выстрелы при такой
большой инструментальной погрешности особого смысла нет —
лучше поработать над точностью приборов и методикой измерений.

2.6 Обработка косвенных измерений

Косвенными называют измерения, полученные в результате расчётов,
использующих результаты прямых (то есть «непосредственных»)
измерений физических величин. Сформулируем основные правила пересчёта
погрешностей при косвенных измерениях.

2.6.1 Случай одной переменной

Пусть в эксперименте измеряется величина x, а её «наилучшее»
(в некотором смысле) значение равно x⋆ и оно известно с
погрешностью σx. После чего с помощью известной функции
вычисляется величина y=f⁢(x).

В качестве «наилучшего» приближения для y используем значение функции
при «наилучшем» x:

Найдём величину погрешности σy. Обозначая отклонение измеряемой
величины как Δ⁢x=x-x⋆, и пользуясь определением производной,
при условии, что функция y⁢(x) — гладкая
вблизи x≈x⋆, запишем

где f′≡d⁢yd⁢x — производная фукнции f⁢(x), взятая в точке
x⋆. Возведём полученное в квадрат, проведём усреднение
(σy2=⟨Δ⁢y2⟩,
σx2=⟨Δ⁢x2⟩), и затем снова извлечём
корень. В результате получим


Пример. Для степенной функции
y=A⁢xn имеем σy=n⁢A⁢xn-1⁢σx, откуда



σyy=n⁢σxx,или  εy=n⁢εx,

то есть относительная погрешность степенной функции возрастает пропорционально
показателю степени n.

Пример. Для y=1/x имеем ε1/x=εx
— при обращении величины сохраняется её относительная
погрешность.

Упражнение. Найдите погрешность логарифма y=ln⁡x, если известны x
и σx.

Упражнение. Найдите погрешность показательной функции y=ax,
если известны x и σx. Коэффициент a задан точно.

2.6.2 Случай многих переменных

Пусть величина u вычисляется по измеренным значениям нескольких
различных независимых физических величин x, y, …
на основе известного закона u=f⁢(x,y,…). В качестве
наилучшего значения можно по-прежнему взять значение функции f
при наилучших значениях измеряемых параметров:

Для нахождения погрешности σu воспользуемся свойством,
известным из математического анализа, — малые приращения гладких
функции многих переменных складываются линейно, то есть справедлив
принцип суперпозиции малых приращений:

где символом fx′≡∂⁡f∂⁡x обозначена
частная производная функции f по переменной x —
то есть обычная производная f по x, взятая при условии, что
все остальные аргументы (кроме x) считаются постоянными параметрами.
Тогда пользуясь формулой для нахождения дисперсии суммы независимых
величин (2.7), получим соотношение, позволяющее вычислять
погрешности косвенных измерений для произвольной функции
u=f⁢(x,y,…):

σu2=fx′⁣2⁢σx2+fy′⁣2⁢σy2+… (2.11)

Это и есть искомая общая формула пересчёта погрешностей при косвенных
измерениях.

Отметим, что формулы (2.10) и (2.11) применимы
только если относительные отклонения всех величин малы
(εx,εy,…≪1),
а измерения проводятся вдали от особых точек функции f (производные
fx′, fy′ … не должны обращаться в бесконечность).
Также подчеркнём, что все полученные здесь формулы справедливы только
для независимых переменных x, y, …

Остановимся на некоторых важных частных случаях формулы
(2.11).


Пример. Для суммы (или разности) u=∑i=1nai⁢xi имеем



σu2=∑i=1nai2⁢σxi2.

(2.12)


Пример. Найдём погрешность степенной функции:
u=xα⋅yβ⋅…. Тогда нетрудно получить,
что



σu2u2=α2⁢σx2x2+β2⁢σy2y2+…

или через относительные погрешности



εu2=α2⁢εx2+β2⁢εy2+…

(2.13)


Пример. Вычислим погрешность произведения и частного: u=x⁢y или u=x/y.
Тогда в обоих случаях имеем



εu2=εx2+εy2,

(2.14)

то есть при умножении или делении относительные погрешности складываются
квадратично.


Пример. Рассмотрим несколько более сложный случай: нахождение угла по его тангенсу



u=arctgyx.

В таком случае, пользуясь тем, что (arctgz)′=11+z2,
где z=y/x, и используя производную сложной функции, находим
ux′=uz′⁢zx′=-yx2+y2,
uy′=uz′⁢zy′=xx2+y2, и наконец



σu2=y2⁢σx2+x2⁢σy2(x2+y2)2.

Упражнение. Найти погрешность вычисления гипотенузы z=x2+y2
прямоугольного треугольника по измеренным катетам x и y.

По итогам данного раздела можно дать следующие практические рекомендации.

  • Как правило, нет смысла увеличивать точность измерения какой-то одной
    величины, если другие величины, используемые в расчётах, остаются
    измеренными относительно грубо — всё равно итоговая погрешность
    скорее всего будет определяться самым неточным измерением. Поэтому
    все измерения имеет смысл проводить примерно с одной и той же
    относительной погрешностью
    .

  • При этом, как следует из (2.13), особое внимание
    следует уделять измерению величин, возводимых при расчётах в степени
    с большими показателями. А при сложных функциональных зависимостях
    имеет смысл детально проанализировать структуру формулы
    (2.11):
    если вклад от некоторой величины в общую погрешность мал, нет смысла
    гнаться за высокой точностью её измерения, и наоборот, точность некоторых
    измерений может оказаться критически важной.

  • Следует избегать измерения малых величин как разности двух близких
    значений (например, толщины стенки цилиндра как разности внутреннего
    и внешнего радиусов): если u=x-y, то абсолютная погрешность
    σu=σx2+σy2
    меняется мало, однако относительная погрешность
    εu=σux-y
    может оказаться неприемлемо большой, если x≈y.

Two Types of Experimental Error

Andrew Brookes / Getty Images

No matter how careful you are, there is always error in a measurement. Error is not a «mistake»—it’s part of the measuring process. In science, measurement error is called experimental error or observational error.

There are two broad classes of observational errors: random error and systematic error. Random error varies unpredictably from one measurement to another, while systematic error has the same value or proportion for every measurement. Random errors are unavoidable, but cluster around the true value. Systematic error can often be avoided by calibrating equipment, but if left uncorrected, can lead to measurements far from the true value.

Key Takeaways

  • Random error causes one measurement to differ slightly from the next. It comes from unpredictable changes during an experiment.
  • Systematic error always affects measurements the same amount or by the same proportion, provided that a reading is taken the same way each time. It is predictable.
  • Random errors cannot be eliminated from an experiment, but most systematic errors can be reduced.

Random Error Example and Causes

If you take multiple measurements, the values cluster around the true value. Thus, random error primarily affects precision. Typically, random error affects the last significant digit of a measurement.

The main reasons for random error are limitations of instruments, environmental factors, and slight variations in procedure. For example:

  • When weighing yourself on a scale, you position yourself slightly differently each time.
  • When taking a volume reading in a flask, you may read the value from a different angle each time.
  • Measuring the mass of a sample on an analytical balance may produce different values as air currents affect the balance or as water enters and leaves the specimen.
  • Measuring your height is affected by minor posture changes.
  • Measuring wind velocity depends on the height and time at which a measurement is taken. Multiple readings must be taken and averaged because gusts and changes in direction affect the value.
  • Readings must be estimated when they fall between marks on a scale or when the thickness of a measurement marking is taken into account.

Because random error always occurs and cannot be predicted, it’s important to take multiple data points and average them to get a sense of the amount of variation and estimate the true value.

Systematic Error Example and Causes

Systematic error is predictable and either constant or else proportional to the measurement. Systematic errors primarily influence a measurement’s accuracy.

Typical causes of systematic error include observational error, imperfect instrument calibration, and environmental interference. For example:

  • Forgetting to tare or zero a balance produces mass measurements that are always «off» by the same amount. An error caused by not setting an instrument to zero prior to its use is called an offset error.
  • Not reading the meniscus at eye level for a volume measurement will always result in an inaccurate reading. The value will be consistently low or high, depending on whether the reading is taken from above or below the mark.
  • Measuring length with a metal ruler will give a different result at a cold temperature than at a hot temperature, due to thermal expansion of the material.
  • An improperly calibrated thermometer may give accurate readings within a certain temperature range, but become inaccurate at higher or lower temperatures.
  • Measured distance is different using a new cloth measuring tape versus an older, stretched one. Proportional errors of this type are called scale factor errors.
  • Drift occurs when successive readings become consistently lower or higher over time. Electronic equipment tends to be susceptible to drift. Many other instruments are affected by (usually positive) drift, as the device warms up.

Once its cause is identified, systematic error may be reduced to an extent. Systematic error can be minimized by routinely calibrating equipment, using controls in experiments, warming up instruments prior to taking readings, and comparing values against standards.

While random errors can be minimized by increasing sample size and averaging data, it’s harder to compensate for systematic error. The best way to avoid systematic error is to be familiar with the limitations of instruments and experienced with their correct use.

Key Takeaways: Random Error vs. Systematic Error

  • The two main types of measurement error are random error and systematic error.
  • Random error causes one measurement to differ slightly from the next. It comes from unpredictable changes during an experiment.
  • Systematic error always affects measurements the same amount or by the same proportion, provided that a reading is taken the same way each time. It is predictable.
  • Random errors cannot be eliminated from an experiment, but most systematic errors may be reduced.

Sources

  • Bland, J. Martin, and Douglas G. Altman (1996). «Statistics Notes: Measurement Error.» BMJ 313.7059: 744.
  • Cochran, W. G. (1968). «Errors of Measurement in Statistics». Technometrics. Taylor & Francis, Ltd. on behalf of American Statistical Association and American Society for Quality. 10: 637–666. doi:10.2307/1267450
  • Dodge, Y. (2003). The Oxford Dictionary of Statistical Terms. OUP. ISBN 0-19-920613-9.
  • Taylor, J. R. (1999). An Introduction to Error Analysis: The Study of Uncertainties in Physical Measurements. University Science Books. p. 94. ISBN 0-935702-75-X.

Разница между случайной и систематической ошибкой

Если ошибка не имеет какой-либо конкретной модели возникновения, она известна как случайная ошибка, которая также известна как несистематическая ошибка, и, следовательно, такие ошибки нельзя предсказать заранее, как неизбежную ошибку, тогда как систематическая ошибка — это ошибка, которая может возникнуть. из-за любой ошибки в измерении прибора ошибка или ошибка в использовании прибора экспериментатором и, следовательно, это ошибка, которой можно избежать.

Основное отличие состоит в том, что случайные ошибки в основном приводят к колебаниям, которые окружают истинное значение из-за трудностей при проведении измерений, тогда как систематические ошибки приводят к предсказуемым, а также постоянным отклонениям от истинного значения из-за проблем с калибровка оборудования.

Независимо от того, насколько осторожны при проведении экспериментов, скорее всего, будет ошибка, называемая экспериментальной ошибкой. Будь то из-за присущих ему проблем, связанных с проблемами с вашим оборудованием, точным выполнением измерений или полным предотвращением ошибки, это практически невозможно.

Чтобы противостоять упомянутой проблеме, ученые стараются изо всех сил классифицировать эти ошибки и пытаться количественно оценить любую неопределенность в измерениях, которые они делают. Выявление разницы между этими ошибками является жизненно важной частью обучения, позволяющего разрабатывать более эффективные эксперименты и пытаться свести к минимуму любые ошибки, которые действительно подкрадываются.

Инфографика случайных и систематических ошибок

Давайте посмотрим основные различия между случайной ошибкой и систематической ошибкой.

Ключевые отличия

Ключевые отличия заключаются в следующем:

  • Случайная ошибка определяет себя как непредсказуемое нарушение, которое возникает в вашем эксперименте из-за неизвестного источника. При этом систематическая ошибка возникает из-за неисправности аппарата, который не построен.
  • Случайная ошибка, как указано в приведенной выше таблице, возникает в обоих направлениях, тогда как систематическая ошибка возникает только в одном направлении. Систематические ошибки возникают из-за встроенной неисправности или ошибки аппарата; следовательно, он всегда дает аналогичную ошибку. Случайная ошибка, как упоминалось ранее, возникает из-за неизвестного источника, поэтому она возникает в любом направлении.
  • Величина систематической ошибки будет оставаться постоянной или неизменной, потому что дефект, который присутствует в ней, встроен внутри устройства, и по сравнению с величиной случайной ошибки он имеет переменную величину.
  • Ошибка 0 и неправильная калибровка прибора вызовут систематическую ошибку. Случайная ошибка возникает из-за параллакса или, как указано ранее в приведенной выше сравнительной таблице, из-за неправильного использования устройства.
  • Случайная погрешность уменьшается или может быть минимизирована путем получения 2 или более показаний одного и того же эксперимента, в то время как систематическая ошибка может быть минимизирована путем тщательного проектирования конструкции устройства.
  • Случайная ошибка сама по себе уникальна и не имеет конкретных типов, тогда как систематическая ошибка может быть разделена на три основных типа: ошибка среды, ошибка прибора и систематическая ошибка.
  • Случайная ошибка не воспроизводится, с другой стороны, систематическая ошибка будет воспроизводимой, потому что дефект, как указано ранее, встроен в структуру устройства.

Сравнительная таблица случайных и систематических ошибок

Основа Случайная ошибка Систематическая ошибка
Основное определение Это ошибки, которые колеблются из-за неопределенности или непредсказуемости, присущей вашему процессу измерения, или различий в величине, которую вы пытаетесь измерить. Это происходит в основном из-за недостатков оборудования, то есть они обычно возникают из-за неправильной калибровки оборудования.
Величина ошибки  Величина ошибки меняется при каждом чтении. Измеренное значение будет либо очень низким, либо очень высоким по сравнению с истинным значением.
Причины 1) Ошибка параллакса

2) Неправильное использование аппарата.

3) Ограничение инструмента, среды и т. Д.

1) Нулевая ошибка

2) Неправильная калибровка

Методы минимизации Повторно снимая показания. 1) За счет улучшения конструкции аппарата.

2) Ошибка нуля может быть уменьшена путем вычитания из ошибки нуля полученного показания.

Направление ошибки Это происходит с обеих сторон Это происходит только в одном направлении.
Подтипы ошибок Подтипов нет. Есть 3 подтипа — a. Инструмент b. Систематическая ошибка c. Среда.
Воспроизводимо ли это Этот вид ошибки не воспроизводится Этот вид ошибки воспроизводится
С точки зрения стоимости Цена — это комбинация стоимости, которая в основном связана с производством. Затраты снижаются, когда они сравниваются со стоимостью с точки зрения стоимости.

Вывод

Случайная ошибка в основном возникает из-за каких-либо нарушений, происходящих в вашем окружении, таких как колебания или перепады давления, температуры или из-за наблюдателя, который может принимать неправильные или неправильные показания. Систематическая ошибка, возможно, также возникает из-за механической конструкции аппарата.

Случайных ошибок по существу нельзя избежать, а систематических ошибок можно избежать. Ученые не могут делать точных масштабов или измерений, какими бы умелыми они ни были.

Систематические ошибки, возможно, трудно обнаружить, и это связано с тем, что все, что вы измеряете, будет неверным или неверным на ту же величину, и вы, возможно, вообще не осознаете, что существует проблема. Перед использованием необходимо правильно откалибровать оборудование, и да, тогда вероятность систематических ошибок будет намного меньше.

  • 2023

Время и Стекло Так выпала Карта HD VKlipe Net

Время и Стекло Так выпала Карта HD VKlipe Net

Оглавление:

  • Ключевые вынос
  • Случайный пример ошибки и причины
  • Пример и причины систематической ошибки
  • Ключевые выводы: случайная ошибка против систематической ошибки
  • источники

Независимо от того, насколько вы осторожны, всегда есть ошибка в измерении. Ошибка не является «ошибкой» — это часть процесса измерения. В науке ошибка измерения называется экспериментальной ошибкой или наблюдательной ошибкой.

Существует два широких класса ошибок наблюдений: случайная ошибка а также систематическая ошибка, Случайная ошибка изменяется непредсказуемо от одного измерения к другому, в то время как систематическая ошибка имеет одинаковое значение или пропорцию для каждого измерения.

Ключевые вынос

  • Случайная ошибка приводит к тому, что одно измерение немного отличается от следующего. Это происходит из-за непредсказуемых изменений во время эксперимента.
  • Систематическая ошибка всегда влияет на результаты измерений в одинаковом или одинаковом соотношении, при условии, что показания проводятся каждый раз одинаково. Это предсказуемо.
  • Случайные ошибки не могут быть устранены из эксперимента, но большинство систематических ошибок могут быть уменьшены.

Случайный пример ошибки и причины

Если вы делаете несколько измерений, значения группируются вокруг истинного значения. Таким образом, случайная ошибка в первую очередь влияет на точность. Как правило, случайная ошибка влияет на последнюю значащую цифру измерения.

Основными причинами случайной ошибки являются ограничения инструментов, факторы окружающей среды и небольшие изменения в процедуре. Например:

  • При взвешивании на весах вы позиционируете себя немного по-разному каждый раз.
  • При измерении объема в колбе вы можете каждый раз читать значение под другим углом.
  • Измерение массы образца на аналитических весах может привести к различным значениям, когда воздушные потоки влияют на весы или когда вода входит в образец и покидает его.
  • Измерение вашего роста зависит от незначительных изменений осанки.
  • Измерение скорости ветра зависит от высоты и времени измерения. Многократные чтения должны быть взяты и усреднены, потому что порывы и изменения в направлении влияют на значение.
  • Показания должны оцениваться, когда они находятся между отметками на шкале или когда учитывается толщина измерительной отметки.

Поскольку случайная ошибка всегда возникает и не может быть предсказана, важно взять несколько точек данных и усреднить их, чтобы получить представление о величине отклонения и оценить истинное значение.

Пример и причины систематической ошибки

Систематическая ошибка предсказуема и либо постоянна, либо пропорциональна измерению. Систематические ошибки в первую очередь влияют на точность измерения.

Типичные причины систематической ошибки включают в себя ошибку наблюдений, несовершенную калибровку прибора и помехи окружающей среды. Например:

  • Если забыть о тарировании или обнулении баланса, получаются измерения массы, которые всегда «выключаются» на одну и ту же величину. Ошибка, вызванная не установкой прибора на ноль до его использования, называется ошибка смещения.
  • Не считывание мениска на уровне глаз для измерения объема всегда приводит к неточным показаниям. Значение будет постоянно низким или высоким, в зависимости от того, взяты ли показания сверху или ниже отметки.
  • Измерение длины с помощью металлической линейки даст другой результат при холодной температуре, чем при высокой температуре, из-за теплового расширения материала.
  • Неправильно откалиброванный термометр может дать точные показания в определенном температурном диапазоне, но стать неточным при более высоких или более низких температурах.
  • Измеренное расстояние отличается при использовании новой измерительной ленты от старой натянутой ленты. Пропорциональные ошибки этого типа называются ошибки масштабного коэффициента.
  • дрейф происходит, когда последовательные чтения становятся последовательно ниже или выше со временем. Электронное оборудование имеет тенденцию быть склонным к дрейфу. Многие другие инструменты подвержены (как правило, положительному) дрейфу, поскольку устройство нагревается.

Как только его причина установлена, систематическая ошибка может быть уменьшена до некоторой степени. Систематическая ошибка может быть сведена к минимуму путем регулярной калибровки оборудования, использования контрольных элементов в экспериментах, прогрева приборов до снятия показаний и сравнения значений со стандартами.

В то время как случайные ошибки могут быть минимизированы путем увеличения размера выборки и усреднения данных, сложнее компенсировать систематическую ошибку. Лучший способ избежать систематической ошибки — это знать ограничения инструментов и иметь опыт их правильного использования.

Ключевые выводы: случайная ошибка против систематической ошибки

  • Двумя основными типами ошибок измерения являются случайная ошибка и систематическая ошибка.
  • Случайная ошибка приводит к тому, что одно измерение немного отличается от следующего. Это происходит из-за непредсказуемых изменений во время эксперимента.
  • Систематическая ошибка всегда влияет на результаты измерений в одинаковом или одинаковом соотношении, при условии, что показания проводятся каждый раз одинаково. Это предсказуемо.
  • Случайные ошибки не могут быть устранены из эксперимента, но большинство систематических ошибок могут быть уменьшены.

источники

  • Блэнд, Дж. Мартин и Дуглас Г. Альтман (1996). «Статистические заметки: ошибка измерения». BMJ 313.7059: 744.
  • Cochran, W.G. (1968). «Ошибки измерения в статистике». Technometrics, Taylor & Francis, Ltd. от имени Американской статистической ассоциации и Американского общества качества. 10: 637–666. DOI: 10,2307 / 1267450
  • Додж Ю. (2003). Оксфордский словарь статистических терминов, ОУП. ISBN 0-19-920613-9.
  • Тейлор, Дж. Р. (1999). Введение в анализ ошибок: изучение неопределенностей в физических измерениях, Университетская Наука Книги. п. 94. ISBN 0-935702-75-X.

Независимо от того, насколько вы осторожны, всегда есть ошибка в измерении. Ошибка не является «ошибкой» — это часть процесса измерения. В науке ошибка измерения называется экспериментальной ошибкой или наблюдательной ошибкой.

Существует два широких класса ошибок наблюдений: случайная ошибка а также систематическая ошибка, Случайная ошибка изменяется непредсказуемо от одного измерения к другому, в то время как систематическая ошибка имеет одинаковое значение или пропорцию для каждого измерения.

Ключевые вынос

  • Случайная ошибка приводит к тому, что одно измерение немного отличается от следующего. Это происходит из-за непредсказуемых изменений во время эксперимента.
  • Систематическая ошибка всегда влияет на результаты измерений в одинаковом или одинаковом соотношении, при условии, что показания проводятся каждый раз одинаково. Это предсказуемо.
  • Случайные ошибки не могут быть устранены из эксперимента, но большинство систематических ошибок могут быть уменьшены.

Случайный пример ошибки и причины

Если вы делаете несколько измерений, значения группируются вокруг истинного значения. Таким образом, случайная ошибка в первую очередь влияет на точность. Как правило, случайная ошибка влияет на последнюю значащую цифру измерения.

Основными причинами случайной ошибки являются ограничения инструментов, факторы окружающей среды и небольшие изменения в процедуре. Например:

  • При взвешивании на весах вы позиционируете себя немного по-разному каждый раз.
  • При измерении объема в колбе вы можете каждый раз читать значение под другим углом.
  • Измерение массы образца на аналитических весах может привести к различным значениям, когда воздушные потоки влияют на весы или когда вода входит в образец и покидает его.
  • Измерение вашего роста зависит от незначительных изменений осанки.
  • Измерение скорости ветра зависит от высоты и времени измерения. Многократные чтения должны быть взяты и усреднены, потому что порывы и изменения в направлении влияют на значение.
  • Показания должны оцениваться, когда они находятся между отметками на шкале или когда учитывается толщина измерительной отметки.

Поскольку случайная ошибка всегда возникает и не может быть предсказана, важно взять несколько точек данных и усреднить их, чтобы получить представление о величине отклонения и оценить истинное значение.

Пример и причины систематической ошибки

Систематическая ошибка предсказуема и либо постоянна, либо пропорциональна измерению. Систематические ошибки в первую очередь влияют на точность измерения.

Типичные причины систематической ошибки включают в себя ошибку наблюдений, несовершенную калибровку прибора и помехи окружающей среды. Например:

  • Если забыть о тарировании или обнулении баланса, получаются измерения массы, которые всегда «выключаются» на одну и ту же величину. Ошибка, вызванная не установкой прибора на ноль до его использования, называется ошибка смещения.
  • Не считывание мениска на уровне глаз для измерения объема всегда приводит к неточным показаниям. Значение будет постоянно низким или высоким, в зависимости от того, взяты ли показания сверху или ниже отметки.
  • Измерение длины с помощью металлической линейки даст другой результат при холодной температуре, чем при высокой температуре, из-за теплового расширения материала.
  • Неправильно откалиброванный термометр может дать точные показания в определенном температурном диапазоне, но стать неточным при более высоких или более низких температурах.
  • Измеренное расстояние отличается при использовании новой измерительной ленты от старой натянутой ленты. Пропорциональные ошибки этого типа называются ошибки масштабного коэффициента.
  • дрейф происходит, когда последовательные чтения становятся последовательно ниже или выше со временем. Электронное оборудование имеет тенденцию быть склонным к дрейфу. Многие другие инструменты подвержены (как правило, положительному) дрейфу, поскольку устройство нагревается.

Как только его причина установлена, систематическая ошибка может быть уменьшена до некоторой степени. Систематическая ошибка может быть сведена к минимуму путем регулярной калибровки оборудования, использования контрольных элементов в экспериментах, прогрева приборов до снятия показаний и сравнения значений со стандартами.

В то время как случайные ошибки могут быть минимизированы путем увеличения размера выборки и усреднения данных, сложнее компенсировать систематическую ошибку. Лучший способ избежать систематической ошибки — это знать ограничения инструментов и иметь опыт их правильного использования.

Ключевые выводы: случайная ошибка против систематической ошибки

  • Двумя основными типами ошибок измерения являются случайная ошибка и систематическая ошибка.
  • Случайная ошибка приводит к тому, что одно измерение немного отличается от следующего. Это происходит из-за непредсказуемых изменений во время эксперимента.
  • Систематическая ошибка всегда влияет на результаты измерений в одинаковом или одинаковом соотношении, при условии, что показания проводятся каждый раз одинаково. Это предсказуемо.
  • Случайные ошибки не могут быть устранены из эксперимента, но большинство систематических ошибок могут быть уменьшены.

источники

  • Блэнд, Дж. Мартин и Дуглас Г. Альтман (1996). «Статистические заметки: ошибка измерения». BMJ 313.7059: 744.
  • Cochran, W.G. (1968). «Ошибки измерения в статистике». Technometrics, Taylor & Francis, Ltd. от имени Американской статистической ассоциации и Американского общества качества. 10: 637–666. DOI: 10,2307 / 1267450
  • Додж Ю. (2003). Оксфордский словарь статистических терминов, ОУП. ISBN 0-19-920613-9.
  • Тейлор, Дж. Р. (1999). Введение в анализ ошибок: изучение неопределенностей в физических измерениях, Университетская Наука Книги. п. 94. ISBN 0-935702-75-X.

Разница между случайной и систематической ошибкой

Если ошибка не имеет какой-либо конкретной модели возникновения, она известна как случайная ошибка, которая также известна как несистематическая ошибка, и, следовательно, такие ошибки нельзя предсказать заранее, как неизбежную ошибку, тогда как систематическая ошибка — это ошибка, которая может возникнуть. из-за любой ошибки в измерении прибора ошибка или ошибка в использовании прибора экспериментатором и, следовательно, это ошибка, которой можно избежать.

Основное отличие состоит в том, что случайные ошибки в основном приводят к колебаниям, которые окружают истинное значение из-за трудностей при проведении измерений, тогда как систематические ошибки приводят к предсказуемым, а также постоянным отклонениям от истинного значения из-за проблем с калибровка оборудования.

Независимо от того, насколько осторожны при проведении экспериментов, скорее всего, будет ошибка, называемая экспериментальной ошибкой. Будь то из-за присущих ему проблем, связанных с проблемами с вашим оборудованием, точным выполнением измерений или полным предотвращением ошибки, это практически невозможно.

Чтобы противостоять упомянутой проблеме, ученые стараются изо всех сил классифицировать эти ошибки и пытаться количественно оценить любую неопределенность в измерениях, которые они делают. Выявление разницы между этими ошибками является жизненно важной частью обучения, позволяющего разрабатывать более эффективные эксперименты и пытаться свести к минимуму любые ошибки, которые действительно подкрадываются.

Инфографика случайных и систематических ошибок

Давайте посмотрим основные различия между случайной ошибкой и систематической ошибкой.

Ключевые отличия

Ключевые отличия заключаются в следующем:

  • Случайная ошибка определяет себя как непредсказуемое нарушение, которое возникает в вашем эксперименте из-за неизвестного источника. При этом систематическая ошибка возникает из-за неисправности аппарата, который не построен.
  • Случайная ошибка, как указано в приведенной выше таблице, возникает в обоих направлениях, тогда как систематическая ошибка возникает только в одном направлении. Систематические ошибки возникают из-за встроенной неисправности или ошибки аппарата; следовательно, он всегда дает аналогичную ошибку. Случайная ошибка, как упоминалось ранее, возникает из-за неизвестного источника, поэтому она возникает в любом направлении.
  • Величина систематической ошибки будет оставаться постоянной или неизменной, потому что дефект, который присутствует в ней, встроен внутри устройства, и по сравнению с величиной случайной ошибки он имеет переменную величину.
  • Ошибка 0 и неправильная калибровка прибора вызовут систематическую ошибку. Случайная ошибка возникает из-за параллакса или, как указано ранее в приведенной выше сравнительной таблице, из-за неправильного использования устройства.
  • Случайная погрешность уменьшается или может быть минимизирована путем получения 2 или более показаний одного и того же эксперимента, в то время как систематическая ошибка может быть минимизирована путем тщательного проектирования конструкции устройства.
  • Случайная ошибка сама по себе уникальна и не имеет конкретных типов, тогда как систематическая ошибка может быть разделена на три основных типа: ошибка среды, ошибка прибора и систематическая ошибка.
  • Случайная ошибка не воспроизводится, с другой стороны, систематическая ошибка будет воспроизводимой, потому что дефект, как указано ранее, встроен в структуру устройства.

Сравнительная таблица случайных и систематических ошибок

Основа Случайная ошибка Систематическая ошибка
Основное определение Это ошибки, которые колеблются из-за неопределенности или непредсказуемости, присущей вашему процессу измерения, или различий в величине, которую вы пытаетесь измерить. Это происходит в основном из-за недостатков оборудования, то есть они обычно возникают из-за неправильной калибровки оборудования.
Величина ошибки  Величина ошибки меняется при каждом чтении. Измеренное значение будет либо очень низким, либо очень высоким по сравнению с истинным значением.
Причины 1) Ошибка параллакса

2) Неправильное использование аппарата.

3) Ограничение инструмента, среды и т. Д.

1) Нулевая ошибка

2) Неправильная калибровка

Методы минимизации Повторно снимая показания. 1) За счет улучшения конструкции аппарата.

2) Ошибка нуля может быть уменьшена путем вычитания из ошибки нуля полученного показания.

Направление ошибки Это происходит с обеих сторон Это происходит только в одном направлении.
Подтипы ошибок Подтипов нет. Есть 3 подтипа — a. Инструмент b. Систематическая ошибка c. Среда.
Воспроизводимо ли это Этот вид ошибки не воспроизводится Этот вид ошибки воспроизводится
С точки зрения стоимости Цена — это комбинация стоимости, которая в основном связана с производством. Затраты снижаются, когда они сравниваются со стоимостью с точки зрения стоимости.

Вывод

Случайная ошибка в основном возникает из-за каких-либо нарушений, происходящих в вашем окружении, таких как колебания или перепады давления, температуры или из-за наблюдателя, который может принимать неправильные или неправильные показания. Систематическая ошибка, возможно, также возникает из-за механической конструкции аппарата.

Случайных ошибок по существу нельзя избежать, а систематических ошибок можно избежать. Ученые не могут делать точных масштабов или измерений, какими бы умелыми они ни были.

Систематические ошибки, возможно, трудно обнаружить, и это связано с тем, что все, что вы измеряете, будет неверным или неверным на ту же величину, и вы, возможно, вообще не осознаете, что существует проблема. Перед использованием необходимо правильно откалибровать оборудование, и да, тогда вероятность систематических ошибок будет намного меньше.

Поскольку
выборка охватывает , как правило,
весьма незначительную часть генеральной
совокупности, то следует предполагать,
что будут иметь место различия между
оценкой и характеристикой генеральной
совокупности, которую эта оценка
отображает. Эти различия получили
название ошибок отображения или ошибок
репрезентативности. Ошибки
репрезентативности подразделяются
на два типа : систематические и случайные.

Систематические
ошибки

это постоянное завышение или занижение
значения оценки по сравнению с
характеристикой генеральной совокупности
. Причиной появления систематической
ошибки является несоблюдение принципа
равновероятности попадания каждой
единицы генеральной совокупности в
выборку , то есть выборка формируется
из преимущественно «худших» ( или «
лучших») представителей генеральной
совокупности. Соблюдение принципа
равновозможности попадания каждой
единицы в выборку позволяет полностью
исключить этот тип ошибок .

Случайные
ошибки

это меняющиеся
от выборки к выборке по знаку и величине
различия между оценкой и оцениваемой
характеристикой генеральной совокупности
. Причина возникновения случайных
ошибок- игра случая при формировании
выборки, составляющей лишь часть
генеральной совокупности. Этот тип
ошибок органически присущ выборочному
методу. Исключить их полностью нельзя,
задача состоит в том , чтобы предсказать
их возможную величину и свести их к
минимуму. Порядок связанных в связи
с этим действий вытекает из рассмотрения
трех видов случайных ошибок : конкретной
, средней и предельной.

2.2 Конкретная, средняя и предельная ошибки выборки

2.2.1
Конкретная

ошибка – это ошибка одной проведенной
выборки. Если средняя по этой выборке
(
) является оценкой для генеральной
средней (0
) и, если
предположить, что эта генеральная
средняя нам известна , то разница
=0
и будет
конкретной ошибкой этой выборки. Если
из этой генеральной совокупности
выборку повторим многократно, то каждый
раз получим новую величину конкретной
ошибки :
…,
и так далее.
Относительно этих конкретных ошибок
можно сказать следующее: некоторые из
них будут совпадать между собой по
величине и знаку, то есть имеет место
распределение ошибок, часть из них
будет равна 0, наблюдается совпадение
оценки и параметра генеральной
совокупности;

2.2.2
Средняя ошибка

– это средняя квадратическая из всех
возможных по воле случая конкретных
ошибок оценки :
,
где— величина меняющихся конкретных
ошибок;частота
( вероятность ) встречаемости той или
иной конкретной ошибки. Средняя
ошибка выборки показывает насколько
в среднем можно ошибиться , если на
основе оценки делается суждение о
параметре генеральной совокупности.
Приведенная формула раскрывает
содержание средней ошибки, но она не
может быть использована для практических
расчетов, хотя бы потому, что предполагает
знание параметра генеральной совокупности
, что само по себе исключает необходимость
выборки.

Практические
расчеты средней ошибки оценки
основываются на той предпосылке, что
она ( средняя ошибка ) по сути является
средним квадратическим отклонением
всех возможных значений оценки. Эта
предпосылка позволяет получить алгоритмы
расчета средней ошибки, опирающиеся
на данные одной единственной выборки.
В частности средняя ошибка выборочной
средней может быть установлена на
основе следующих рассуждений. Имеется
выборка (
,) состоящая изединиц. По выборке в качестве оценки
генеральной средней определена
выборочная средняя. Каждое значение(,) , стоящее под знаком суммы, следует
рассматривать как независимую случайную
величину, поскольку при бесконечном
повторении выборки первая, вторая и
т.д. единицы могут принимать любые
значения из присутствующих в генеральной
совокупности. СледовательноПоскольку , как известно, дисперсия
суммы независимых случайных величин
равна сумме дисперсий , то.
Отсюда следует, что средняя ошибка для
выборочной средней будет равнаяи находится она в обратной зависимости
от численности выборки ( через корень
квадратный из нее ) и в прямой от среднего
квадратического отклонения признака
в генеральной совокупности. Это логично,
поскольку выборочная средняя является
состоятельной оценкой для генеральной
средней и по мере увеличения численности
выборки приближается по своему значению
к оцениваемому параметру генеральной
совокупности. Прямая зависимость
средней ошибки от колеблемости признака
обусловлена тем, что чем больше
изменчивость признака в генеральной
совокупности, тем сложнее на основе
выборки построить адекватную модель
генеральной совокупности. На практике
среднее квадратическое отклонение
признака по генеральной совокупности
заменяется его оценкой по выборке, и
тогда формула для расчета средней
ошибки выборочной средней приобретает
вид:,
при этом учитывая смещенность
выборочной дисперсии,
выборочное среднее квадратическое
отклонение рассчитывается по формуле=. Так как символомn
обозначена численность выборки. ,то
в знаменателе при расчете среднего
квадратического отклонения должна
использоваться не численность выборки
( n
), а так называемое число степеней
свободы (n-1).
Под числом степеней свободы понимается
число единиц в совокупности, которые
могут свободно варьировать ( изменяться
), если по совокупности определена
какая-либо характеристика. В нашем
случае , поскольку по выборке определена
ее средняя, свободно варьировать могут

единицы.

В
таблице 2.2 приведены формулы для
расчета средних ошибок различных
выборочных оценок . Как видно из этой
таблицы, величина средней ошибки по
всем оценкам находится в обратной связи
с численностью выборки и в прямой с
колеблемостью. Это можно сказать и
относительно средней ошибки выборочной
доли ( частости ). Под корнем стоит
дисперсия альтернативного признака,
установленная по выборке (
)

Приведенные
в таблице 2.2 формулы относятся к так
называемому случайному , повторному
отбору единиц в выборку. При других
способах отбора , о которых речь пойдет
ниже, формулы будут несколько
видоизменяться.

Таблица
2.2

Формулы для
расчета средних ошибок выборочных
оценок

Выборочные
оценки

Формулы
для расчета средней ошибки выборочной
оценки

Выборочная
средняя (
)

Выборочная
дисперсия
(
)

Выборочное
среднее квадратическое отклонение
( s
)

Выборочная
доля (w
)

2.2.3
Предельная ошибка выборки

Знание оценки и ее средней ошибки в
ряде случаев совершенно недостаточно
. Например , при использовании гормонов
при кормлении животных знать только
средний размер неразложившихся их
вредных остатков и среднюю ошибку,
значит подвергать потребителей продукции
серьезной опасности. Здесь настоятельно
напрашивается необходимость определения
максимальной ( предельной
ошибки
).
При использовании выборочного метода
предельная ошибка устанавливается не
в виде конкретной величины , а виде
равных границ

(
интервалов) в ту и другую сторону от
значения оценки.

Определение
границ предельной ошибки основывается
на особенностях распределения конкретных
ошибок . Для так называемых больших
выборок, численность которых более 30
единиц (
)
, конкретные ошибки распределяются в
соответствии с нормальным законом
распределения; при малых выборках () конкретные ошибки распределяются
в соответствии с законом распределения
Госсета

(
Стьюдента ). Применительно к конкретным
ошибкам выборочной средней функция
нормального распределения имеет
вид:
,
где— плотность вероятности появления тех
или иных значений,
при условии, что,
гдевыборочные средние;
генеральная средняя,— средняя ошибка для выборочной
средней. Поскольку средняя ошибка
()
является величиной постоянной, то в
соответствии с нормальным законом
распределяются конкретные ошибки,
выраженные в долях средней ошибки, или
так называемых нормированных отклонениях
.

Взяв
интеграл функции нормального
распределения, можно установить
вероятность того , что ошибка будет
заключена в некотором интервале
изменения t
и вероятность того, что ошибка выйдет
за пределы этого интервала ( обратное
событие ). Например , вероятность того,
что ошибка не превысит половину средней
ошибки ( в ту и другую сторону от
генеральной средней ) составляет
0,3829, что ошибка будет заключена в
пределах одной средней ошибки — 0,6827,
2-х средних ошибок -0,9545 и так далее.

Взаимосвязь
между уровнем вероятности и интервалом
изменения t
( а в конечном счете интервалом
изменения ошибки ) позволяет подойти
к определению интервала ( или границ )
предельной ошибки, увязав его величину
с вероятностью осуществления..
Вероятность осуществления -это
вероятность того, что ошибка будет
находится в некотором интервале.
Вероятность осуществления будет
«доверительной» в том случае, если
противоположное событие ( ошибка будет
находится вне интервала ) имеет такую
вероятность появления, которой можно
пренебречь. Поэтому доверительный
уровень вероятности устанавливают,
как правило, не ниже 0,90 (вероятность
противоположного события равна 0,10 ).
Чем больше негативных последствий
имеет появление ошибок вне установленного
интервала, тем выше должен быть
доверительный уровень вероятности (
0,95; 0,99 ; 0,999 и так далее ).

Выбрав
доверительный уровень вероятности
по таблице интеграла вероятности
нормального распределения, следует
найти соответствующее значение t,
а затем используя выражение
=определить интервал предельной ошибки.
Смысл полученной величины в следующем
– с принятым доверительным уровнем
вероятности предельная ошибка выборочной
средней не превысит величину.

Для
установления границ предельной ошибки
на основе больших выборок для других
оценок ( дисперсии, среднего квадратического
отклонения, доли и так далее ) используется
выше рассмотренный подход, с учетом
того, что для определения средней
ошибки для каждой оценки используется
свой алгоритм.

Что
касается малых выборок () то, как уже говорилось, распределение
ошибок оценок соответствует в этом
случае распределениюt
— Стьюдента. Особенность этого
распределения состоит в том, что в
качестве параметра в нем , наряду с
ошибкой, присутствует численность
выборки ,вернее не численность выборки,
а число степеней свободы
При увеличении численности выборки
распределениеt-Стьюдента
приближается к нормальному, а при
эти распределения практически совпадают.
Сопоставляя значения величиныt-Стьюдента
и t
— нормального распределения при одной
и той же доверительной вероятности
можно сказать , что величина t-Стьюдента
всегда больше t
— нормального распределения, причем,
различия возрастают с уменьшением
численности выборки и с повышением
доверительного уровня вероятности.
Следовательно, при использовании малых
выборок имеют место по сравнению с
выборками большими , более широкие
границы предельной ошибки, причем , эти
границы расширяются с уменьшением
численности выборки и повышением
доверительного уровня вероятности.

Вопросы для
повторения

6-1.Какова
природа конкретной, средней и предельной
ошибок ?

6-2.Как
соблюсти принцип равновероятности
каждой единицы попасть в выборку при
выборочном устном опросе студентов ?

6-3 Каков источник
систематической ошибки ?

6-4.Какова
вероятность появления ошибки в 2.5 раза
превышающей среднюю?

6-5.Какие
различия в знаках ( + , — ) имеют
систематические и случайные ошибки?

6-6.Каковы основные
пути уменьшения средней и предельной
ошибки ?

6-7.При какой
выборочной доле имеет место ее наибольшая
ошибка ?

6-8.При какой доле
признака имеет место ее наименьшая
ошибка 7

6-9.При
каких выборках ( больших или малых )
при прочих равных условиях имеет место
большая предельная ошибка ?

Резюме по
модульной единице 2

Использование
выборочного метода неизбежно сопряжено
с появлением ошибок. Случайный характер
этих ошибок, нормальный или t
— Стьюдента закон их распределения
позволяет определить их средний и
предельный размер и видеть пути их
снижения

Модульная
единица 3 Типовые задачи решаемые на
основе выборочного метода

Соседние файлы в предмете [НЕСОРТИРОВАННОЕ]

  • #
  • #
  • #
  • #
  • #
  • #
  • #
  • #
  • #
  • #
  • #

Понравилась статья? Поделить с друзьями:
  • Система торможения бмв ошибка
  • Систематическая ошибка линейки
  • Систематическая ошибка социология
  • Система снижения токсичности рено каптур ошибка как исправить
  • Систематическая ошибка выжившего это